You are on page 1of 91

MATHEMATICS

Class - XII Booklet - I

Content

Sr.No. Chapter Name Pg.no.

1. Function 1 - 55
2. Inverse Trigonometric Function 56 - 90
Page 1

FUNCTION 1
UNIT

 Theory

 Drill Exercises

 Solved Examples

 Exercises

 Single Correct Type Questions

 Multiple Correct Type Questions

 Comprehension Type Questions

 Matrix Match Questions.

 Integer Type Questions

 Previous Years Questions

 Answer Key
Page 2 FUNCTION
INTERVALS
The set of numbers between any two real numbers is called interval. The following
are the types of interval.
(i) Closed Interval: [a, b] = {x : a  x  b}
(ii) Open Interval: (a, b) or ]a, b[ = {x : a < x < b}
(iii) Semi open or semi closed interval:
[a, b[ or [a, b) = {x: a  x < b}
]a, b] or (a, b] = {x: a < x  b}
Inequalities
The following are some very useful points to remember:
1. a  b  either a < b or a = b
2. a < b and b < c  a < c
3. a < b  –a > –b i.e., inequality sign reverses if both sides are multiplied by a negative
number
4. a < b and c < d  a + c < b + d and a – d < b – c.  c  R
5. a < b  ma < mb if m > 0 and ma > mb if m < 0
6. 0 < a < b  ar < br if r > 0 and ar > br if r < 0

 1
7.  a  a   2 for a > 0 and equality holds for a = 1
 

 1
8.  a  a   – 2 for a < 0 and equality holds for a = –1
 

DRILL - I
Solve the following Inequations:
2x  3
1. 3 2. (x – 1)2 (x + 4) < 0
3x  5 
3. (2x + 1) (x – 3) (x + 7) < 0 4. x2 + 6x – 27 > 0 and x2 – 3x – 4 < 0,
5. x2 – 1  0 and x2 – x – 2  0 6. (x – 1) ( (3 – x) (x – 2)2 > 0.
2
( x  1)( x  2) 1  3x 2
7.  0. 8. 2
 0.
1 x 2 x  21x  40

x 2 1 x 2  6x  7
9. 2
 . 10. 2
 2.
x 1 2 x 1

14x 9x  30 x 1
11.   0. 12.  x.
x 1 x  4 x 1

(2  x 2 )(x  3) 3 4 3
x  3x  2 x
2

13. 2
 0. 14. 2
 0.
( x  1)( x  3x  4) x  x  30

2( x  4) 1 1 1 1
15.  . 16.   .
( x  1)( x  7) x  2 x  2 x 1 x
2 2
 x 6
17. x(ex – 1)(x + 2)(x – 3)2  0 18. 32x  2.3x  32(x6)  0
Page 3 FUNCTION

(x  1)3 (x  2)4 (x  3)5 (x  6)


19. Let f (x) = Find the set of volues of x when.
x 2 (x  7)3

(a) f (x) > 0 (b) f (x)  0 (c) f (x) < 0 (d) f (x)  0

Definition of Function
Let A and B be two non–empty sets. Then a function ‘f ’ from set A to set B is a rule
which associates elements of set A to elements of set B such that
An element of set A is associated to a unique element in set B.
All elements of set A are associated to element in set B.
Terms such as “map” (or mapping), “correspondence” are used as synonyms for function.
f
If f is a function from a set A to set B, then we write f : A  B or A   B. which is
read as f is a function from A to B or f maps A to B.

A f B

x1 y1
x2 y2
x3 y3
xn ym

Total number of maping from set A to set B = mn


Ex.1 Let A = {2, 4, 6, 8} and B = {s, t, u, v, w} be two sets and let f 1, f2, f3 and f4 be rules
associating elements of A to elements of B as shown in the following figures.

s f2 s
2 f1
t 2 t
4 u 4 u
6 v 6 v
8 w 8 w

f3 s f4 s
2 t 2
4 u 4 t
6 v u
w 6 v
8 8 w
Now see that f1 is not function from set A to set B, since there is an element 6  A
which is not associated to any element of B, but f2 and f3 are the function from A to B,
because under f2 and f3 each elements in A is associated to a unique element in B.
But f4 is not function from A to B because an elements 8  A is associated to two
elements u and w in B.
Domain:Set A is called domain of f i.e. Set of those elements from which functions is
to be defined.
Co–Domain : Here set B is called co–domain of function.
Range : Set of images of each element in A, is called range of f.
Note: Range  Co–domain
Real Valued Function :
All those functions of which domain and Co–Domain are subsets of R are called real
valued functions. In this case for a given function we have to find domain and Range.
Page 4 FUNCTION
Bounded Function :
A function ‘f’ is said to be bounded if | f ( x) |  m , for some finite ‘m’ for every x in
Domain of f.
Some Elementary Functions
General Exponential Function
If a > 0, a  1 then the function defined by f(x) = ax, x  R is called an Exponential
Function with base a.
Y
–x
y=4 –x x x
–x y = 10 y = 10 y = 4 y = 2 x
y=2 Domain : R

+
a>1 Range : R

Nature : one-one

0<a<1

O X

Logarithmic Function
If a > 0, a  1, then the function y = loga x, x  R+(set of positive real numbers) is
called the logarithmic Function with base a.

Y
y=log2x
y=log 4x
y=log10x
Domain : R+
X Range : R
O
Nature : one-one
y=log1/10x
y=log1/4x
y=log1/2x

Polynomial Function :
If a function is defined by f(x) = a0 xn + a1 xn – 1 + a2 xn – 2 + ..........+ an– 1 x + an , where n
is a non negative integer and a0, a1, a2, .........an are real numbers and a 0  0 , then
f is called a polynomial function of degree n.
Note :
(A) A polynomial of degree one with no constant term is called an odd linear function.
i.e. f(x) = ax, a  0 .
(B) There are two polynomial functions, satisfying the relation; f(x).f(1/x) = f(x) + f(1/x).
They are
(i) f(x) = xn + 1 (ii) f(x) = 1 – xn, where n is a positive integer
(C) f(x) = c and c  0 is a polynomial of degree zero.
(D) f(x) = 0 is a polynomial but degree not defined.
Note : Function given in (C) and (D) are also called constant function.
Page 5 FUNCTION
Algebraic Function :
y is an algebraic function of x, if it is a function that satisfies an algebraic equation of
the form, P0 (x)y n  P1(x)y n1  ...........  Pn1(x)y  Pn (x)  0 where n is a positive integer
and P0 (x),P1(x).......... are Polynomials in x. e.g. y = |x| is an algebraic function, since
2 2
it satisfies the equation y – x = 0.
Note that all polynomial functions are Algebraic but not the converse. A function that
is not algebraic is called Transcendental Function.
Rational Function
The function which can be written as the quotient of two polynomial functions is said
to be a rational function.
If P(x) = a0 + a1x + a2x2 + . . . + anxn
Q (x) = b0 + b1x + b2x2 + . . . + bmxm

P(x)
be two polynomial functions then a function f defined by f(x) = is a rational
Q(x)
function of x

7x 4  x 2  2
e.g., f(x) = is a rational function which is defined for all real values of x
x 2  4x  3
except 1 and 3.
Identity Function
A map f : R  R is said to be an identity function, iff f(x) = x,  x  R.
The identity function is sometimes also called the function x
Domain of the identity function = R
Range of the identity function = R

Modulus Function
y = –x, x < 0 y y = x, x > 0
  x, x  0
f(x) = |x| = 
 x, x  0
O x
x
Domain : R, Range : [0,  )
It is an even continuous and many one function
y
Graph is symmetrical with respect to y–axis.

y
Signum Function

1, x  0 y = 1, x > 0
 x
0, x  0 O
f(x) =  x
1, x  0
 y = –1, x < 0
y
Domain ; R, Range; {–1, 0, 1}. It is a many one and discontinuous function
Ex.1 (i) Determine the values of x satisfying the equality:
|(x2 + 4x + 9) + (2x –3)| = |x2 + 4x + 9| + |2x –3|;

1 x f (x ).f ( x 2 ) 1
(ii) If f(x) = , show that  .
1 x 1  [f ( x)]2 2
Page 6 FUNCTION
Sol. (i) The equality |a + b| = |a| + |b| is valid if and only if both summands have the
same sign. Since x2 + 4x + 9 = (x + 2)2 + 5 > 0 at any values of x, the equality is
satisfied at those values of x at which 2x – 3  0, i.e., at x  3/2.
1 x2 1  x2
(ii) f(x2) = 
1  x 2 1  x 1  x 

1 x  1  x2 1  x2
1 x  1 x 1 x 2
f(x).f(x 2 )     1  x  2
1 x 1
    
1  [f(x)]2 2
1  x 
2 22
1  x   1  x  2  2x 2
1 2 2
1  x  1  x 
DRILL - II
Modulus Function
1. Solve the following equations.
a. | x + 2 | = 2(3 – x) b. | 3x – 2 | + x = 11

c. xx2  2 d. x2 + | x – 1 | = 1

x x2
e. |x–1|+|x–2|+|x–3|  6 f.  | x |
x 1 | x 1 |

2. Find the set of all solution of equation 2| y |  | 2 y 1  1 |  2 y 1  1

3. Solve the following inequalitites


a. | 5 – 2x | < 1 b. | 3x – 2  5 |  2
c. |x–2|  |x+4| d. x2 - | x | - 2  0
e. | x2 – 4x | < 5 f. | x2 – 5x | < 6
g. | x2 – 2x | < x h. | x2 – 2x -3 | < 3x – 3
4. Find the solution of | x2 – 1 + sin x | = | x2 – 1 | + | sin x |, belonging to the interval
[– 2  , 2  ].

Greatest Integer Function


If f(x) = k;  x  [ k, k  1) , where k is any integer, then f is called greatest integer
function usually denoted by f(x) = [x]

Ex.: [3.7] = 3, [–3  2] = – 4, [5] = 5 etc.


Properties of Greatest Integer Function
1. x – 1 < [x]  x
4. [x + n] = [x] + n, where n  I

 0; x  I
2. [x] + [– x] = 
 1; x  I
3. [x] = n  n  x  n  1
[x]  n  x  n, n  I
[x]  n  x < n + 1, n  I
Page 7 FUNCTION

Fractional Part of x
 x  1, x  [1,0)
f(x) = x – [x], x  R i.e., f(x) = {x} =  x, x  [0,1)
 x  1, x  [1,2)

Domain : R, Range : [0, 1), Nature : Many one
This is a periodic function with period 1. It is discontinuous at all integers.
Properties Of Fractional Part Of x
1. x = [x] + {x} where [.] and {.} denotes the integral and fractional part of x
respectively.
0; x  I
2. {x} + {– x} =  .
1; x  I

DRILL - III
Greatest Integer Function
(Where [ ] is G.I.F. & { } is Fractional Part)

3 3 1  3 2   3 99 
1. Find the value of           ........    .
 4   4 100   4 100   4 100 
2. Given y = 2[x] + 3 and y = 3[x – 2] + 5, then find [x + y].
3. If [x]2 – 9[x] + 8  0, then find x.
4. If 3[x]2 – 7[x] + 4 > 0, then find x.
5. Solve 4{x} = x + [x].
6. Solve {x + 1} + 2x = 4[x + 1] – 6.
2000
x  r
7. Find [x] + 
r 1 2000
(r  integer).

1 1 1
8. Solve   {x} 
[ x] [ 2 x] 3
9. Find the real solution of [x] + [5x] + [10x] + [20x] = 36 K + 35., K  Integer.

Algebra of Functions
Given function f : D1  R and g : D2  R, we describe function f + g, f – g, fg and
f/g as follows
f + g : D  R is a function defined by (f + g)(x) = f(x) + g(x),
f – g : D  R is a function defined by (f – g) (x) = f(x) – g(x)
fg : D  R is a function defined by (fg) (x) = f(x) g(x)
f f f(x)
: C  R is a function defined by   (x) = g(x) ,g(x)  0 ,
g g
where D = D1  D2 and C = {x  D : g (x)  0}

Ex.2 Find the domain of following functions:


1 1
(i) f(x)  2sin x
 x2 (ii) f(x) = sin–1 4  x2
log10  x  1
Page 8 FUNCTION
(iii) f(x) = ln (– 2 + 3x – x2)
Sol. (i) For f(x) to be defined – 1  x  1,
x  2  0 i.e., x  2 ,
x + 1 > 0 i.e., x > – 1 and
x + 1  1 i.e., x  0
so, domain of f : (– 1, 0)  (0, 1]
(ii) f(x) = sin–1 4  x2
for f(x) to be defined 0  4 – x2  1  x2 – 4  0 and x2 – 3  0

 x  [ – 2, 2] and x  (–  , – 3 ]   3,  
 x   2,  3    3,2

so domain (f) : x    2,  3    3 , 2 
(iii) f(x) = ln (–2 + 3x – x2)
for f(x) to be defined – 2 + 3x – x2 > 0  x2 – 3x + 2 < 0
 (x – 1) (x – 2) < 0  x  (1, 2)
so domain (f) : x  (1, 2)

DRILL - IV
Find the domain of the functions:

1
1. f(x) = 2. f (x) = x 1 + 6x
x2

1
3. f(x) = 2  2x  x 2 4. f(x) = x  [x]

x3
5. f(x) = log |log x| 6. f(x) =
(x  1) x 2  4

1 log 2  x  3 
7. y  x 2 8. f(x) =
log10 (1  x) x 2  3x  2

x 1
9. f(x) = 10. f(x) = x 2  3x  2 
2 2
x  3x  2 x  3x  4

2  5x  x 2 
11. f(x) = 2 + log10 (x3 – x) 12. f(x) = log1 / 2  
x 4  4 

13. 10x + 10y = 10 14. f(x) = 3  2 x  21 x

 x  2x  1
15. log (   x  0 16. f(x) =
x2 )
|x|  x  3x 2  2 x
3

17. f(x) = log4 [log5 {log3 (18x – x2 – 77)}] 18. f(x) = log(1 – log10 (x2 –5x + 16))
2
( x  2)
19. f(x) = log(x + 1) (x2 –3x + 2) 20. f(x) = 6
4x  83  52  2 2( x 1)
Page 9 FUNCTION

  6  
21. f(x) = log2  – log1/2 1  4   2 
  x 

22. f(x) = 4
log 0.2 x 3  log 0.2 x 3 log 0.2 0.0016 x   36
1 2x 
23. f(x) = log 10   24. f(x) = log(2x–5) (x2 –3x –10)
 x3 
1 1
25. f(x) = 26. f(x) =
x x x  x (From 26 to 32; [ ] is G.I.F.)
1  x 
27. f(x) = 28. f ( x )   x 2   1  3   x 2 
2  x 

1 1
29. f(x) = | x  1|  |7  x |  6 30. f(x) = | x | 1  5
31. f(x) = log4 (log3 (log2 (x2 –2x + 3 ) –log (2x – 1)))
2

   4  x2 
32. f(x) = [sin x] cos 33. f(x) = sin  log  
[ x  1]   1 x 
  

34. f(x) = log 2 (sin x ) 35. f x   sin x  1

Methods of determining range :


(i) Representing x in terms of y
If y = f(x), try to express as x = g(y), then domain of g(y) represents possible values
of y, which is range of f(x).
(ii) Graphical Method :
The set of y– coordinates of the graph of a function is the range.
(iii) Using monotonocity : Many of the functions are monotonic increasing or monotonic
decreasing. In case of monotonic continuous functions the minimum and maximum
values lie at end points of domain. Some of the common function which are
increasing or decreasing in the interval where they are continuous is as under.

For monotonic increasing functions in [a, b]


Page 10 FUNCTION

y
y = f(x)

x
x=a x=b
(i) f ' (x)  0
(ii) range is [f(a), f(b)]
for monotonic decreasing functions in [a, b]
y

y = f(x)

x
x= a x= b
(i) f ' (x)  0
(ii) range is [f(b), f(a)]

Ex.3 Find the range of the following functions:


1
(i) f(x) = (ii) f(x) = x2 – 7x + 5
8  3 sin x
(iii) f(x) = log2 (log1/2 (x2 + 4x + 4))
1
Sol. (i) f(x) = . We know that –1  sin x  1
8  3 sin x
 1 1
 – 3  3 sin x  3  5  8 – 3 sin x  11  Range (f) = 11 , 5 
 

2
 7 29  29 
(ii) f(x) = x – 7x + 5  f(x) =  x   
2
 Range (f) =   ,
 2 4  4 
(iii) f(x) = log2 (log1/2 (x2 + 4x + 4))
since 0 < log1/2 (x2 + 4x + 4) <   x  Domain (f)
 –  < log2 (log1/2 (x2 + 4x + 4)) < 
Range (f) = (– ,  )

DRILL - V
Find the range of the functions:

|x  3|
1. f(x) = 2. f(x) = 9 – 7 sin x
x 3
3. f(x) = 2 + x – [x–3] 4. f(x) = x2 – 6x + 7

x
5. f (x)  3 C x 6. f (x) 
1  x2

 x2  e  1
7. f(x) = ln  2  8. f(x) = x2 + 2
 x 1  x 1
9. y = (x - 1)2 + 4 10. y = 4 - |x + 1|
Page 11 FUNCTION
11. y = 4 x – 2x + 1 12. y = sin2 x + cos4 x

1
13. y = [x]+[-x]; ([ ] is G.I.F.) 14. y=
2  cos 5 x

x2
15. y= 16. y = x2 + x + 1
x4 1
17. y = 4x + 2x + 1 18. y = {x} + {-x}; ({ } is fractional part)

1
19. y=
x  x ; ({ } is fractional part) 20. y = 3sinx + 4cosx

  e x  ex
21. y = 3sinx + 4cos  x   +5 22. y=
 3 e x  ex

x  [ x]
23. y = log e(3x2 – 4x + 5) 24. y = 1  [ x]  x ; ([ ] is G.I.F.)

25. y = log2 ( x  4  6  x ) 26. y= x 1  5  x

 sin x  cos x  3 2   2 
27. f(x) = log 2   28. y = 3 cos   x 2 
 2   9 

 1 
29. f(x)  n  
 sec x 

30.
 2tan x 
If f   =
 
1  cos 2 x sec 2 x  2 tan x 
find the domain and Range of f(x)
 1  tan2 x  2
1
31. y= sin x   cos x  ; ([ ] is G.I.F.) 32. y = n cos(sin x)

Equality of two functions :


Two function f and g are said to be equal functions, if and only if
(i) domain of f = domain of g (ii) Range of f = Range of g
(iii) f(x) = g(x);  x  their domain
Ex.4 If A = {1, 2}, B = {10, 13}, f : A  B, f(x) = x2 + 9 and g = A  B, g(x) = 3x + 7,
then f = g because domains and Range of both f and g are same also
f(1) = 10 = g(1) ; f(2) = 13 = g(2)

DRILL - VI
Equal and Identical Function
1. Which pair of function are equal or identical?

x2 1 x
(a) f(x) = x, g(x) = (b) f (x)  ,g(x)  2
x x x

(c) f (x)  log e e x , g(x)  e log e x (d) f (x)  loge x 2 , g(x)  2  log e x
Page 12 FUNCTION

(e) f (x)  log e x 3 , g(x)  3  loge x (f) f (x)  x,g(x)  x 2

(g) f (x)  1, g(x)  sin 2 x  cos 2 x

(h) f (x)  sgn(x 2  3x  4), g(x)  e{x} ([.] is G.I.F. and { } is fractional part)

1  cos 2x
(i) f (x)  , g(x)  tan x (j) f (x)  ln(1  x)  ln (1  x), g(x)  ln(1  x 2 )
1  cos 2x

2. (a) Let f(x) = log x 2 25 and g(x) = logx 5 Then f(x) = g(x) holds. Then find the interval
for x.
(b) Let A = {1, 2} B = {3, 6} and f : A  B given by f(x) = x2 + 2 and g:A  B given by
g(x)=3x. Find whether equal or not.

Even and Odd Functions :


If f : X  Y is a real valued function such that for all x  D   x  D (where D = domain
of f ) and if f(–x) = f(x) for every x  D then f is said to be an even function and if f(–x)=
– f(x) then f is said to be an odd function. Even functions are symmetric about the y–
axis (i.e. if (x, y) lies on the curve, then (–x, y) also lies on the curve, and odd
functions are symmetric about the origin (i.e. (x, y) lies on the curve, then (–x, –y)
also lies on the curve.
Remarks
Every function defined in symmetric interval D(i.e. x  D   x  D ) can be expressed
 f(x)  f( x)   f(x)  f(x) 
as a sum of an even and an odd function. f(x) =   .
 2   2 

 f(x)  f( x)   f(x)  f( x) 


Let h(x) =   and g(x) =   . It can now easily be shown that h(x) is
 2   2 
even and g(x) is odd.
The first derivative of an even function is an odd function and vice – versa.
If x  0, x  Domain of f, then for odd function f(x) which is continuous at x = 0, f(0) = 0,
i.e. if for a function f (0)  0 , then that function can not be odd. It follows that for a
differentiable even function f (0)  0 i.e. if for a differentiable function f (0)  0 then
the function cannot be even.
Ex.5 Determine the nature of the following function for even and odd:
 a x 1
 2 
(i) f(x) = log  x  x  1  (ii) f(x) = x  x 
a  1 
 

Sol. (i) f(x) = log ( x  x 2  1)  f(–x) = log (  x  x 2  1)

 1 
= log  2
 = log (x + x2  1 )
–1
= –log (x + x 2  1 ) = – f(x)
 x  x 1 
So, f(x) is an odd function

 a x 1
(ii) We have f(x) = x  x 

 a 1
Page 13 FUNCTION

 1 
 a x  1   x 1   1 a x   a x 1
    x  a    x    x x 
 f(– x) = – x  a x  1  1 a x   a  1  = f(x)
   1     
 x 1
a 
So, f(x) is an even function.

Ex.6 If f is an even function defined in the interval [– 5, 5], find four real values of x
 x 1 
satisfying the equation f(x) = f  .
 x  2
Sol. Since f is an even function, f(– x) = f(x).

 x 1   x 1  x 1 x 1
 Now f  x  2   f ( x )  f  x  2   f (  x )   x or =–x
    x2 x2
x2 + x – 1 = 0 or x2 + 3x + 1 = 0

 1 5 3 5
x = or x = .
2 2

DRILL - VII
1. Determine the nature of the following functions for even and odd.
(a) f(x) = sin x + cos x (b) f(x) = x2 – |x|

x x
(c) f(x) = sin(log (x + x 2  1 )) (d) f(x) = + 1
x
e 1 2

x(sin x  tan x)
(e) f(x) = x  n ; n  I ([ ] is G.I.F.)
x   1
  2
 

1 x 
(f) f(x) = log   (g) f(x) = x sin2x – x3
1 x 
(h) f(x) = sinx – cosx
2. Find out whether the following functions is even or odd.

(a) f(x) = 4 – 2x4 + sin2 x (b) f(x) = 1 x  x 2  1 x  x 2


x
(1  2 x ) 2  1 x 
(c) f(x) =
2x
(d) f(x) =  log
0
e 
1 x 
dx

3. Let f:R  R be a function given by f(x+y) + f(x-y) = 2f(x) f(y) ;  x, y  R. and f(0)  0.
Prove that f(x) is an even function.
4. If f(x + y) = f(x).f(y);  real x,y and f(0)  0. Then prove that the function

f x 
h(x) = 2 is an even function.
1   f(x)
Page 14 FUNCTION

5. Represents each of the following functions as the sum of an even and odd function.
(a) f(x) = ax (b) f(x) = (1+x)100
(c) f(x) = sin 2x + tan x + cosx/2 (d) f(x) = aex
6. Let f(x) = ex + sin x be defined on the interval [-4,0]. Find the odd and even extension
of f(x) in the interval [-4,4].
7. Let the function f(x) = x2 + x + sin x – cos x + ln(1 + |x|) be defined over the interval
[0,1]. Find the odd and even extensions of f(x) in the interval in [-1, 1].

Periodic Function
A function f(x) is said to be periodic function if, there exists a fixed positive real
number T independent of x, such that, f(x + T) = f(x);  x  Domain & x + T domain.
T is called one of the period of the function
In other words, a function is said to be periodic function if its each value is repeated
after a definite interval.
Here the least positive value of T (independent of x) is called the fundamental period
of the function.
Clearly f(x) = f(x + T) = f(x + 2T) = f(x + 3T) = . . .
For example :
(A) sin x, cos x, sec x and cosec x are periodic functions with period 2  .
(B) tan x and cot x are periodic functions with period  .
(C) |sinx|, |cosx|, |tanx|, |cotx|, |secx|, |cosecx| are periodic functions with
period  .
(D) sinnx, cosnx, secnx, cosecnx are periodic functions with period 2  or  according
as n is odd or even.
(E) tannx and cotnx are periodic function with period  whether n is odd or even.
Properties of Periodic Function :
If f(x) is periodic with period T, then
(i) f(x  c) is periodic with period T.
(ii) f(x)  c is periodic with period T.
(iii) c.f(x) is periodic with period T.

T
(iv) f(ax + b) has period |a| , i.e., period is affected only by coefficient of x where; a, b,

c, are constants with a, b  0.


Note : All periodic functions can be analyzed over an interval of one period within the
domain as the same pattern shall be repetitive over the entire domain.

Ex.7 Find the period of f(x) = |sin x| + | cos x|


Sol. |sin x| has period  , |cos x| has period 
Hence, according to the rule of LCM, period of f(x) must be  .

  
But |sin   x | = |cosx| and |cos (  x)| = |sin x|
2  2

 
Since   , period of f(x) is
2 2
Page 15 FUNCTION
Ex.8 If f(x) = sin x + cos ax is a periodic function, show that a is a rational number.
2 2
Sol. Period of sin x = 2  = and period of cos ax =
1 a

2 2
 Period of sin x + cosax = L.C.M of and
1 a

LC.M of 2 and 2 2
= H.C.F. of 1and a =  where  is the H.C.F. of 1 and a.
Here 1 is rational number. H.C.F. is possible between rational and rational number.
So ‘a’ is also rational number.

DRILL - VIII
1. Find the periods
(a) cos4 x (b) sin3 x

(c) cos x (d) cos x

(e) x  [x] ; ([ ] is G.I.F.)


2. Find the period of
(a) f(x) = sin4 x + cos4 x (b) f(x) = tan 3x + cos 5x/2
(c) f(x) = cos x + {x} (d) f(x) = cos(cos x) + cos(sin x)

1  sin x sin x 
(e) f(x) =    (f) f(x) = 5 sin 3x – 7 sin8 x
2  cos x cos x 

(g) f(x) = cos (|sinx| – |cosx|) (h) f (x)  sin(sin x)  e{3x} ; ({ } is fractional part)

x x
(i) f (x)  sin  sin (j) f(x) = sinx + {x}; ({ } is fractional part)
3 4
3. Find the period of the function

(a) f(x)  ex [x]|cos x||cos 2x|....|cos nx|; ([ ] is G.I.F.)

 x   x 
(b) f(x) = sin x + tan x/2 + sin x/22 + tan x/23 + ..... + sin  n 1  + tan  n 
2  2 

 5x 
4 (a) for what integral value of n, is 3  period of the function cos(nx).sin  ?
 n 
(b) Find the period of [x] + [x + 1/3] + [x + 2/3] – 3x + 10 ; ([ ] is G.I.F.)
5. Let f(x) be a function and K be a + ve real no. such that f(x + k) + f(x) = 0;  x  R
prove that f(x) is periodic with period 2k.

6. If a function satisfies the equation f(x + 1) + f(x –1) = 3 f(x) ;  x  R prove that f(x) is
periodic function. Also find its period.
Page 16 FUNCTION

Classification of Function
(i) One–One Function (Injective) :
If each element in the domain of a function has a distinct image in the co–domain
the function is said to be one–one function and is also known as Injective Function.

A f B

x1 y1
x2 y2
x3 y3
xn ym

n(A)  n(B)

 m Pn ; m  n
Total number of one to one functions =  0 ; m  n

Proof : x1 can take m images
x2 can take (m – 1) images
x3 can take (m – 2) images
........................................
........................................
xn can take (m – n + 1) images
Total number of ways = m . (m – 1) . (m – 2) --------- (m – n + 1)
m! m
= (m  n)!  Pn

e.g. f : R  R+ given by y = ex
g : R  R, g(x) = 3x – 7
are one – one functions.
or, f : A  B is one – one
 a  b  f(a)  f(b) for all a, b  A
 f(a) = f(b)  a = b for all a, b  A
(ii) Many–One Function :
If there are two or more than two elements of domain having the same image then
f(x) is called Many – One function.
Total number of many-one function = Total number of function – Total number of one-
m
one function = mn –  Pn ; m  n
0 ; mn
e.g. f : R  R+ f(x) = x2 + 4
g : R  R+ g(x) = x8 + x4 +x2 + 4
Both functions are many one
If the graph of y = f(x) is given and a line parallel to x–axis cuts the curve at more than
one point then function is many one.
or, f : A  B is a many – one function if there exist x, y  A such that x  y but f(x) =
f(y). e.g y = sin x, y = cos x, y = tan x, y = x2, y = x4, . . . . . are many one functions.
Page 17 FUNCTION

(iii) Onto Function (Surjective) :


Let f : X  Y be a function. If each element in the co–domain Y has at least one pre–
image in the domain X i.e. Range f = Co domain, then f is called onto.
A f B

x1 y1
x2 y2
x3 y3
xn ym

n(A)  n(B)

m m r m n
 (1) . Cr .r ; m  n
 r 1
Total number of onto function =  m! ; mn
 0 ; mn

Onto function is also called surjective and if function be both one–one and onto then
function is called Bijective.
or, f : A  B is a surjection iff for each b  B, a  A such that f(A) = b .
e.g. If f : R+  R is defined by y = log2x, then f(x) is Onto function.
(iv) Into Function :
If there exist one or more than one element in the Co–domain Y which is not an
image of any element in the domain X. Then f is into.
In other words f : A  B is an into function if it is not an onto function.
Total number of into function = Total number of function – Total number of onto
function
e.g. Let f : R  R is defined by y = x2 + 1, then f(x) is an into function. But when
f : R  R+ is defined by y = x2 + 1, then f(x) is not into function.
(v) one-one onto function (bijective) :
If A and B are finite sets and f : A  B is a bijection.
Then, function is one-one function and onto function both.

A f B

x1 y1
x2 y2
x3 y3
xn yn

n(A) = n (B)
x1 can take n images
x2 can take (n – 1) images
x3 can take (n – 2) images
........................................
........................................
xn can take 1 image
Total number of bijection from A  B  n!
Page 18 FUNCTION

Note :
dy
1. If domain of f(x) is continuous and > 0,  x in domain then f is One – One, where
dx
equality exist at discrete point.
dy
2. If domain of f(x) is continuous and < 0,  x in domain then f is One – One, where
dx
equality exist at discrete point.
3. If a continuous function f(x) which has either local minima or local maxima or both
then f(x) will be Many – One
4. Every even function is Many – One
5. Every periodic function is Many – One
Ex.9 (a) f : R  R f(x) = 2x + sin x, prove that f is bijective function
n ot e t h at f ' (x) > 0; V x  R
y

x   ; y   and x  –  ; y  – 
& f(0) = 0  one–one onto  bijective
(b) prove that f : R  R f(x) = 2|x| – 2–x is /manyone into

0 if x  0

2|x| – 2–x =  x
2  2 x if x  0

a many one into


(c) Let f: {x, y, z}  {a, b, c} be a one–one function. It is known that only one of
these statements is true and the remaining two are false.
(i) f(x)  b (ii) f(y) = b ; (iii) f(z)  a find f (x):

Ex.10 Let f : (–  ,  )  [2,  ) be a function defined by f(x) = x 2  2a  a 2 , a  R. Find a for


which f is onto.
Sol. For f to be onto range of the function should be [2,  ). So, a2 – 2a = 4
 a=1  5.
DRILL - IX
Examine the bijectivity of the function:

x2
1. f : R  R , f(x) = x |x| 2. f : R  R , f(x) =
1  x2
3. f : R  R, f(x) = x + |x| 4. f : [–1,1]  [–1, 1], f(x) = sin (  /2) x
5. f : R  R, f(x) = [x] ; [ ] is G.I.F. 6. f : R  R, f(x) = x3 + 3x2 + 12x – 2 sin x
7. (a) f : R  R, f(x) = 2x +5 (b) f : R+  R+, f(x) = x, x  0
 
(c) f : R    ,  , f(x) = sin x
 2 2 
Page 19 FUNCTION

8. Let f(x) = ax3 + bx2 + cx + d sin x, then find the condition that f(x) is always one – one.
9. Find the set of values of a for which the function f : R  R given by
f(x) = x3+(a + 2)x2 + 3ax + 5 is one – one.
10. A mapping is selected at random from the set of all mappings of the
set A = {1,2,3…………n} into itself. Then find the number of one - one mappings.
11. A mapping is selected at random from the set of all mappings of the
set A = {1,2,3…………… n} into itself. Then find the no. of mappings from A to A.
12. Find the no. of surjection from A = {1,2,3…………… n} to B = {a,b} for n  2.

x 2  6x  8
13. A function f:R  R is defined by f(x) = . Find the interval of values of 
  6x  8x 2
for which f is onto. If the function one to one for  = 3? Justify your answer.
14. Show that the function f : R  R defined by f(x) = 3x3 + 5;  x  R is a bijection.

x 1
15. Let A = R – {2} and B = R – {1}. If f : A  B is mapping defined by f(x) = . Show that
x2
f is bijective.

 
16. Let f : x  y be a function defined by f(x) = a sin  x   + b cos x + c. If f is a bijection.
 4
Find the sets x and y.
17. Find the no. of surjections from A = {1,2,3,…………………n}, n  3 to B ={a,b,c}.

Composite Function :
Let f: X  Y and g: Y  Z be two functions and D is the set of values of x such that
if x  X, then f(x)  Y. If D   , then the function h defined on D by h(x) = g{f(x)} is
called composite function of g and f and is denoted by gof. It is also called function of
a function.
Note : Domain of gof is D which is a subset of X (the domain of f ). Range of gof is a
subset of the range of g. If D = X, then f(X)  Y.
Pictorially gof(x) can be viewed as under

Note that gof(x) exists only for those x when range of f(x) is a subset of domain of g(x).
graphically it can be represented below
X f Y g Z

x
f(x)
g (f(x))

h h = gof
Properties of Composite Functions :
(a) In general gof  fog (i.e. not commutative)
(b) The composition of functions are associative i.e. if three functions f, g, h
are such that fo(goh) and (fog)oh are defined, then fo(goh) = (fog)oh.
Page 20 FUNCTION

1
Ex.11 If f(x) = x2 +1, g(x) = , then find (fog) (x) and (gof) (x).
x 1
Sol. Given, f(x) = x2 + 1 .....(1)
1
g(x) = .....(2)
x 1
 1 
Now (fog) (x) = f(g(x)) = f   = f(z),
 x 1
1
where z =
x 1
= z2 + 1 [ f(x)  x 2  1]
2
 1  1
=   1  1
 x 1 (x  1)2
Note: Domain of fog(x) is x  R  {1}
(gof) (x) = g(f(x)) = g(x2 + 1) = g(u),
1 1 1
where u = x2 + 1 =  2  2
u 1 x 1 1 x
Note: Domain of gof(x) is x  R  {0}

2  x, if x  0
Ex.12 If f(x) =  , then find (fof)(x).
2  x, if x  0
Sol. f(x) = 2 + |x|
ff(x) = 2 + |f(x)|
= 2 + 2 + |x|
= 4 + |x|
 2log10 x  2 
Ex.13 (a) If f(x) = log100x   and g(x) = {x}. If the function (fog)(x) exists then
 x 
find the range of g(x).
 1   1 1 
Range of g(x)   0,  , 
 100   100 10 
1
Hint: (i) 100 x > 0 & 100 x  1  x 
100
1 1
(ii) x > 0 and log10x + 1 < 0  0 < x < &x 
10 100
(fog)(x) exists  range of g(x)  domain of f (x)

DRILL - X

x
1. If the function f : R  R be given by f(x) = x2 + 2 and g: R  R be given by g(x) = .
x 1
Find fog and gof.
2. If the mapping f and g are given by f(x) = {(1, 2), (3, 5), (4, 1)} and g = {(2, 3), (5, 1), (1, 3)}
then write down pairs in the mapping fog and gof.
3. If f(x) = [x] and g(x) = cos(  x), then find the range of gof.
Page 21 FUNCTION

4. If f (x) = e3x and g(x) =  nx, x > 0, then find (fog)(x).


x
5. If f(x) = , then find (fofof)(x).
1  x2

1  x 0  x  2
6. (a) Let f(x) =  . Find (fof)x.
3  x 2  x  3

x 2  1 x  0  x  11/3 x 1


(b) if f(x) =  2 , g(x) =  1/2 compute (gof)x.
x  1 x  0  x  1 x 1

    5
7. f(x) = sin2 x + sin2  x   + cos x cos  x   and g   =1. Then find (gof)x.
 3  3  4

| x  1| x  1 x  2 x  0
8. Let f(x) =  and g(x) =  . Then find
1  x x  1 x  3 x  0

(a) (f + g)x (b) (fg)x


9. If f(x) = -1 + |x - 1| , -1  x  3 and g(x) = 2 - |x + 1| , -2  x  2 then calculate (fog)x
and (gof)x and also draw their graph.
10. If f(x) = -1 + |x - 2|, 0  x  4 and g(x) = 2 - |x|, -1  x  3. Find the (gof)x and their
graph also.
11. Let f:R  R be a function given by f(x) = ax + b. ;  x  R. Find the consts. a and b
such that (fof) = I (I being the identity function of R).

 1 x 0

12. If f(x) = x3 – 6x2 + 11x – 6 ;  x  R and g(x) = 0 x  0 then draw the graph of
 1 x  0

(gof)x.

Inverse Function
If f : X  Y be a function defined by y = f(x) such that f is both one–one and onto, then
there exists a unique function g : Y  X such that for each y  Y, g ( y)  x . The function
g so defined is called the inverse of f and denoted by f –1.Also f is the inverse of g and
the two functions f and g are said to be inverse of each other.

f (f 1 ( x ))  x ,  x  Y and f 1(f(x))  x,  x  X
–1
Note that f and f are symmetric about the line y = x.

X f Y

x1 y1
x2 y2
x3 y3
xm yn

f–1
Page 22 FUNCTION
Method of Finding Inverse of a Function :
1. If you are asked to check whether the given function y = f(x) is invertible, you need to
check that y = f(x) is one–one and onto.
2. If you are asked to find the inverse of a bijective function f(x), you do the following : if
f –1 be the inverse of ‘f ’ , then f(f –1(x)) = x . Apply the formula of f on f –1(x) and use of
the above identity to solve for f –1(x).

Some standard functions given below along with their inverse functions
–1
(i) f : [0,  )  [0,  ) f : [0,  )  [0,  )
defined by f(x) = x2 defined by f – 1
(x) = x

     
(ii) f :   ,   [–1, 1] f –1
: [–1, 1]    , 
 2 2  2 2
– 1
defined by f(x) = sinx defined by f (x) = sin–1x

(iii) f : [0,  ]  [–1, 1] f – 1 : [–1, 1]  [0,  ]


defined by f(x) = cosx defined by f – 1 (x) = cos–1x

Ex.14 Find the inverse of the function f(x) = n ( x 2  3x  1); x  [1, 3] assuming it to be an onto
function.
Sol. Given f(x) = n(x 2  3x  1)

2x  3
 f (x)  2
 0;  x  [1, 3]
(x  3x  1)
Which is a strictly increasing function. Thus f(x) is injective, given that f(x) is onto.
Hence the given function f(x) is invertible.
Now f ( f–1(x)) = x  n((f 1(x))2  3(f 1(x))  1)  x

 (f 1(x))2  3(f 1(x))  1  ex  0

1 3  9  4.1(1  e x ) 3  (5  4e x )
 f (x)  
2 2
3  (5  4ex ) 3  (5  4ex )
 f–1(x)  (as f 1(x)  [1, 3] ) Hence f–1(x) 
2 2
 x, x 1
 2
Ex.15 Find the inverse of the function f(x) =  x , 1  x  4 .

8 x, x4

 x, x 1
 2
x , 1 x  4
Sol. Given f(x) = 

8 x, x4

Let f(x) = y  x = f–1(y) .......(i)


Page 23 FUNCTION

 
 y, y 1  y, y 1
 
 x   y, 1  y  4  f–1(y) =  y, 1  y  16 [From (i)]
 y2  y2
 , y2  ,
4 y  16
 64 64  64

 x, x 1

Hence f–1(x) =  x, 1  x  16
 x2
 , x  16
 64

DRILL - XI
1
1. f : [1,  )  [2,  ), ƒ(x) = x + then find f–1(x).
x
2. f:[2,  ,)  (–  ,, 4], f(x) = x(4 – x) then find f–1 (x).
3. Find the inverse of the function, If possible.
(a) f(x) = 3x – 5

1
(b) f:[1,  )  [2,  ) f(x) = x 
x
(c) f(x) = x3 + 3 (assume bijective)

(d) y = loga (x+ x 2  1 ),(a > 1)(assuming onto)

(e) y = sin-1(x/3); x  [-3, 3]

e x  e x
4. Let f : R  R be defined by f(x) = . Is f(x) invertible if so, find its inverse.
2
5. Let f : [1/2,  )  [3/4,  ) where f(x) = x2 –x +1. Find the inverse of f(x). Hence, solve
1 3
the equation x2-x+1 =  x ?
2 4
6. (a) Let a function f : R  R be defined by f(x) = x–[x] ;  x  R. Is the function
invertible?
(b) Let f : R  R be defined by f(x) = cos(5x + 2). Is f invertible? Justify your answer.

1
7. Let g(x) be the inverse of f(x) and f '(x) = . Then find g '(x) in terms of g(x).
1 x3
Real Valued Function
If x, y are independent variables, then:
(i) f (xy) = f (x) + f (y)  f (x) = k ln x or f (x) = 0.

(ii) f (xy) = f (x). f (y)  f (x) = x n, n  R

(iii) f (x + y) = f (x). f (y)  f (x) = a kx.

(iv) f (x + y) = f (x) + f (y)  f(x) = kx, where k is a constant.


Page 24 FUNCTION
DRILL - XII

1 1
1. If f(x) = x  . Prove that [f (x)]3 = 3 f   + f(x3).
x x

1   x  
2. f(x) = cos (log x), then prove that f  x  f  y   2 f  y   f  xy    0 .
   

1
3. If f(x) = , find f(f(f(x))) and draw its graph.
1 x

1 1
4. If for non zero x, a f(x) + b f   = -5, where a  b, then find f(x).
x x

1 1
5. If f(x) be a polynomial function satisfying f(x). f   = f(x) + f   and f(4) = 65, then
x x
find f(6).

1
6. If 3f(x) – f   = x for x > 0, find f(ex).
x
7. Let g(x) is a polynomial function satisfying g(x) g(y) = g(x) + g(y) + g(xy)-2;  x, y  R,
if g(2) = 5, then find g(3).
m

8. If f(x) satisfies the relation f(x + y) = f(x) + f(y) ;  x, y  R, and f(1) = 5. Find  f x ,
x=1

also prove that f(x) is odd.


9. If a function f : R  R is given by f(x + y) = f(x) + f(y) ;  x, y  R and f(1)=a. Find
n

 f r .
r=1

9x
10. Let f(x) = show that f(x) + f(1 - x) = 1 and hence, evaluate
9x  3
 1   2   3   1995 
f  + f  + f  + …….. + f  .
 1996   1996   1996   1996 

4x
11. If f : R  R is given by f(x) = x ;  x  R. Prove that f(x) + f(1 - x) = 1 Hence prove
4 2
 1   2   1996 
that f  + f  + …….. + f   = 998.
 1997   1997   1997 
12. Consider a real valued function f(x) satisfying 2f (xy) = (f(x))y + (f(y))x ;  x, y  R and
n
n 1
f(1) = a where a  1. Prove that (a  1)  f (i)  a
i 1
a .

x
13. Let f1(x) = + 10;  x  R and fn(x) = f1  f n1 ( x)  for n  2, then find fn(x).
3
14. If f : R  R be a function defined by f(x + y) = f(x) + f(y);  x, y  R.
Show that f(x) = xf(1);  x  R.
Page 25 FUNCTION
15. Determine all real valued of functions satisfying functional relation

 1  2(1  2 x )
f(x) + f   = ;  x  R – {0 , 1}
1 x  x(1  x)
16. Let g : R  R be given by g(x) = 4x + 3 if gn(x) = gogog ……. og(x), show that
gn(x) = 4nx + 4n-1, if g-n(x) denotes the inverse of gn(x),
prove that g-n(x) = 4-n(x) + (4-n - 1);  x  R.
17. Let n be a +ve integer and define f(n) = 1! + 2! + 3! +….+ n!. Find polynomials P(x) and
Q(x) such that f(n + 2) = Q(n) f(n) + P(n) f(n + 1) ;  n  1.
18. Let f be a function from the set of +ve integers to be set of real numbers i.e. f : N  R,
such that
(i) f(1) = 1
(ii) f(1) + 2f(2) + 3f(3) + ……….. + nf(n) = n(n+1) f(n) for n  2.
Then find the value of f(1994).
19. If f be a function defined on the set of non –ve integers and taking values in the
same set given that

x  f ( x) 
(i) x – f(x) = 19    90  for all non –ve integers.
19   90 
(ii) 1900 < f(1990) < 2000 find the possible values of f(1990).
(Where [ ] denotes G.I.F.)
Elementry Transformations of Graphs
(i) Drawing the graph of y = |f(x)| from the known graph of y = f(x)
|f(x)| = f(x) if f(x)  0 and |f(x)| = – f(x) if f(x) < 0. It means that the graph of f(x) and
|f(x)| would coincide if f(x)  0 and the portions where f(x) < 0 would get inverted in
the upward direction.
The above figure would make the procedure clear.

y = |f(x)|

(ii) Drawing the graph of y = f(|x|) from the known graph of y = f(x)
It is clear that, f(|x|) = f(x),  x  0 and f(|x|) = f(–x),  x < 0. Thus f(|x|) would be an
even function. Graphs of f(|x|) and f(x) would be identical in the first and the fourth
quadrants (as x  0) and as such the graph of f(|x|) would be symmetrical about the
y–axis (as (|x|) is even).

The figure would make the procedure clear.


Page 26 FUNCTION

(iii) Drawing the graph of |y| = f(x) from the known graph of y = f(x)
Clearly |y|  0. If f(x) < 0, graph of |y| = f(x) would not exist. And if f(x)  0, |y| = f(x)
would give y = f(x). Hence graph of |y| = f(x) would exist only in the regions where
f(x)  0 and will be reflected about x–axis only in those regions. Regions where f(x) <
0 will be neglected.
Full lines show the graph of |y| = f(x) and dotted lines depict the corresponding
graph of y = f(x).

(iv) Drawing the graph of y = f(x + a), a  R from the known graph of y = f(x)

Let us take any point x0  domain of f(x), and set x + a = x0 or x = x0 – a. a > 0


 x < x0, and a < 0  x > x0. That mean x0 and x0 – a would give us same abscissa for
f(x) and f(x + a) respectively. As such for a > 0, graph of f(x + a) can be obtained simply
by translating the graph of f(x) in the negative x–direction through a distance ‘a’
units. If a < 0, graph of f(x + a) can be obtained by translating the graph of f(x) in the
positive x–direction through a distance a units.
Accordingly the graph of f(x) + b can be obtained by translating the graph of f(x) either
in the positive y–axis direction (if b > 0) or in the negative y–axis direction (if b < 0),
through a distance |b| units.
(v) Drawing the graph of y = a f(x) from the known graph of y = f(x)

It is clear that the corresponding points (points with same x coordinates) would have
their ordinates in the ratio of 1 : a.
(vi) Drawing the graph of y = f(ax) from the known graph of y = f(x)
Y

y = f(ax)
a>1 y = f(x)
y = f(ax), 0 < a < 1

(0, f(0))

x0
Let us take any point x0  domain of f(x). Let ax = x0 or x =
a
Clearly if 0 < a < 1 then x > x0 and f(x) will stretch by 1/a units against y–axis, and
if a > 1, x < x0, then f(x) will compress by ‘a’ units against y–axis.
Page 27 FUNCTION
Y
(vii) Drawing the graph of y = f–1 (x) from the known graph of y = f(x)
For drawing the graph of y = f–1(x) we have to first (0, /2)
(1,  / 2)
y=x
of all find the interval in which the function is
(/2, 1)
bijective (invertible). Then take the reflection of (– /2, 0)
(0, 1)
X
O(1, 0)
y = f(x) (within the invertible region) about the line (0, –1)
( /2, 0)
 / 2, 1)
y = x.
The reflected part would give us the graph of y = f–1 (x).
e.g. let us draw the graph of y = sin–1 x. We know that
  
y = f(x) = sin x is invertible If f :   ,    1,1 the inverse mapping would be
 2 2
  
  , 
f–1 : [–1, 1]
 2 2
Ex.16 Draw the graphs of the following functions:
(i) y = |sin x|, x  [0, 2  ] (ii) y = sin |x|, x  [– 2  , 2  ]
(iii) |y| = sinx, x  [– 2  , 2  ] (iv) |y| = –sinx, x  [– 2  , 2  ]

1 1
Sol. (i) O 2 (ii) -2 O 2
-1

1 1
(iii)  2  O 2 (iv)  2  O 2
-1 -1
Page 28 FUNCTION

SOLVED EXAMPLES
 x 2  4x  3, x  3 g x   x  3, x4
Ex.1 Let f  x    ,    2 .
 x  4, x  3  x  2x  2, x  4

f
Describe the function g .
Sol. We redefine the function f(x) and g(x) in the intervals as shown below:
 x 2  4x  3, x3

f  x    x  4, 3x4
 x  4, x4


 x  1, x3
 x  3, x3 

g  x    x  3, 3x4 f x  x4

 g x  , x  x  4.
 x 2  2x  2, x4  x 3

 x4
 x 2 , x4
 2x  2
2
Ex.2 Find the Domain of the following functions: f(x) = log [(1.25) 1 x – (0.4096)1+ x]

1 x 2
Sol. f(x) = log  (1.25)  (0.4096)1 x  = logu (let)

4
5 4096  4 
1.25 = and 0.4096   
4 10000  5 
log u defined for u > 0
1 x2 4(1 x )
5 5
      1 – x2 > – 4 (1 + x)
 4 4
 1 – x2 + 4 + 4x > 0
– x2 + 4x + 5 > 0
x2 – 4x – 5 < 0
(x + 1) (x – 5) < 0
x  (– 1, 5)

Ex.3 Find the range of f(x) = a 2 cos2 x  b2 sin2 x  (a2 sin2 x  b 2 cos2 x) , b > a

Sol. Since f(x) = (a2 cos2 x  b2 sin2 x)  (a2 sin2 x  b 2 cos2 x)

Let y = a 2 cos2 x  b2 sin2 x  (a 2  b2 )  (a 2 cos2 x  b 2 sin2 x)


Let  = a2 cos2x + b2sin2x
1
 = [(a2 + b2) + (a2 - b2) cos2x]  y =  + (a2  b2 )  
2
 y2 =  + (a2 + b2) -  + 2 (a2  b 2 )   2
2
1 2 1 
2 2
 y = (a + b ) + 2 2 (a  b2 )2   (a 2  b2 )   
4 2 
Page 29 FUNCTION

2 2
a2  b2
2
Maximum value of y = 2(a + b ) when  =
2
2
Minimum value of y = a 2  b2  2ab when  = b

 range y  [(a + b), 2(a 2  b2 ) ] (since y cannot be negative)

Ex.4 Find the range of the function f(x) = cosx  sin x  


(sin2 x  sin2  )

Sol. y = cos x  sin x  (sin2 x  sin2  ) 


dividing by cos2x we get
y sec2x = tanx + 2 4 2 2 2
tan2 x  sec 2 x sin2   y sec x – 2y tanx sec x – sin  sec x = 0
 y2 (1 + tan2x) – 2y tanx – sin2  = 0 [ sec2x  0]
y2 tan2x – 2y tan x + y2 – sin2  = 0
tan x is real we have D  0
4y2 – 4y2 (y2 – sin2  )  0
4y2 (1 – y2 + sin2  )  0  y2  1 + sin2  , – 1  sin2   y  1  sin2 

 Range of f(x) =   1  sin  , 1  sin  


2 2

Ex.5 If a, b  R be fixed positive numbers such that


f(a + x) = b + [b3 + 1 – 3b2. f(x) + 3b{f(x)}2 – {f(x)}3]1/3
for all x  R then prove that f(x) is a periodic function.
Sol. Here, {f(a + x) – b}3
= b3 + 1 – 3b2 f(x) + 3b{f(x)}2 – {f(x)}3
= 1 – [{f(x)}3 – 3b. {f(x)}2 + 3b2. f(x) – b3]
= 1 – {f(x) – b}3
 {f(a + x) – b}3 + {f(x) – b}3 = 1 .... . . (i)
This is true for all x.
Putting a + x for x in (i), we get,
{f(2a + x) – b}3 + {f(a + x) – b}3 = 1............ (ii)
Subtracting (i) from (ii)
{f(2a + x) – b}3 – {f(x) – b}3 = 0 or {f(2a + x) – b}3 = {f(x) – b}3
or f(2a + x) – b = f(x) – b or f(2a + x) = f(x)
 f(x) is a periodic function.
Ex.6 Let f(x, y) be a periodic function, satisfying the condition f(x, y) = f(2x + 2y, 2y -2x);
x
 x, y  R and let g(x) be a function defined as g(x) = f(2 , 0). Prove that g(x) is
periodic function and find its period.
Sol. f(x, y) = f(2x + 2y , 2y - 2x)
= f(2(2x + 2y) + 2(2y - 2x), 2(2y - 2x) - 2(2x + 2y)
f(x, y) = f(8y, -8x) = f(8(-8x), - 8 (8y)) = f(-64x, -64y) = f(64 (64x), 64(64 (y))
= f(212x, 212y)
f(x, 0) = f(212x, 0)
f(2y, 0) = f(212 . 2y, 0) = f(212+y, 0)
 g(y) = g(y + 12)
Hence g(x) is periodic and its period is 12.
Page 30 FUNCTION
Ex.7 Find out whether the given function is even, odd or neither
 x|x| , x  1

f(x) = [1  x]  [1  x] , 1  x  1 .
  x|x| , x 1

 x 2 x  1

f  x   1  x   1  x  1  x  1
Sol.
 2
 x x 1

 x 2  x  1  x  1

f   x   1  x   1  x  1   x  1  1  x  1
 2
 x  x  1  x  1
= f(x)
 f  x  is even.
Ex.8 ABCD is a square of side a. A line parallel to the diagonal BD at a distance x from the
vertex A cuts two adjacent sides. Express the area of the segment of the square,
with A at a vertex, as a function of x.
Sol. There are two different situations
a
(i) when x = AP  OA, i.e., x < D F
2 D C C
a
a F O a O P
(ii) When x = AP >OA, i.e., x > but x < 2a P E
2
A B A B
1 a
a
Case (i) ar(  AEF) = x. 2x = x2
2
(PE = PF = AP = x)

Case (ii) ar (ABEFDA) = ar (ABCD) – ar (DCFE)


1
= a2 –
2
.  
2a  x . 2  2a  x  [ CP = 2 a – x]


2 2

= a2 – 2a  x  2 2ax = 2 2ax  x 2  a 2
 the required function f(x) is as follows :

 2 a
x , 0 x
2

f(x)  
2 2ax  x 2  a 2 , a  x  2a .
 2

x 2  3x  6
Ex.9 If f : R  R where f  x   then check whether the function is onto or into.
x2  x  1
2
 3  15
x   
Sol. Here f  x    2 4 , which takes only positive values.
2
 1 3
x   
 2 4
Hence f(x) is into.
Page 31 FUNCTION

1  3  3  1 
Ex.10 Show that the functions f :  ,     ,   and g :  ,     ,   defined as
2  4  4  2 

1 3
f(x) = x2 – x + 1 and g(x) =  x are inverse of each other and hence solve the
2 4

2 1 3
equation x  x  1   x .
2 4

1 1 3
Sol. Given f(x) = x2 – x + 1, x  and g(x) =  x 
2 2 4
1 
Given, f(x) is defined in  ,  
2 
4.1.1  12 3
f(x) is a continuous function and its least value =  (at x = 1/2)
4 4
Also, f(  ) = 
Therefore range of f = [3/4,  )
Thus f : [1/2,  )  [3/4,  )
Domain of g = [3/4,  ) and its range = [1/2,  )
f ' (x) = 2x – 1  0 in [1/2,  ) equality holding only at one point x = 1/2.
Hence f(x) is one–one function from [1/2,  ) onto [3/4,  )
Let y = f(x)  y = x2 – x + 1  x2 – x + (1 – y) = 0

1  1  4(1  y) 1  4y  3 3
 x= = [ x  1/2] = 1/2 + y
2 2 4

1 3
 f–1 (y) =  y
2 4

1 3
 f-1(x) =  x  = g(x)
2 4
Thus f(x) and g(x) are inverse of each other.
Second part:
We know that the curves y = f(x and y = f–1(x) intersect only at the line y = x
 solution of f(x) = f–1(x) are same as those of equation f(x) = x
Now, f(x) = x  x2 – x + 1 = x  x = 1.
Ex.11 Let f : X  [1, 27] be a function defined by f(x) = 5 sinx + 12 cos x + 14. Find the set
X so that f is both one and onto.
Sol. f : X  1,27 

f  x   5sin x  12 cos x  14
Let 5  r cos , 12  r sin 

f  x   r sin  x      14
The function f(x) to be invertible
 / 2  x     / 2
Page 32 FUNCTION

12
 / 2  x  Tan 1  / 2
5
12  12 
  / 2  Tan 1  x  cot 1  
5 5
 x 1
Ex.12 Find all functions f satisfying the identity, f(x) + f   = 1 + x,  x  R-{0, 1}.
 x 

 x 1
Sol. Given f(x) + f   = 1 + x   R - {0, 1} ... (i)
 x 

 x 1 
1  x  1   1  x 1
x 1  x 1  x x 1
Replacing x by  f   f   1  f f   1  .. (ii)
x  x   x 1  x  x   x 1 x
 x 

 1  x 1
Subtracting (ii) from (i), we get f(x) - f  x ... (iii)
 x 1 x

 1 
1  1  1
1  1   x  1 1
 f(x) 1 
Replacing x by ,  f  f   1   f  ...
x 1  x  1   1  x 1  x 1 x 1
 x 1 

SINGLE CORRECT TYPE QUESTIONS


Domain:
1. Domain of the function log|x2 – 9| is
(A) R (B) R– [–3,3] (C) R – {–3,3} (D) None of these
2. The domain of the function f(x) = sin 1/x is -
(A) R – {0} (B) R+ (C) Z (D) R–

 1,when x  Q
3. If f : R  R, f(x) =  , then image set of R under f is -
 1,when x  Q
(A) {1,1} (B) (–1,–1) (C) {1,–1} (D) None of these
2
4. If x is the radius of a circle and f(x) =  x , then domain of f is -
(B) R (B) R+ (C) R¯ (D) R0

x 2  2x  1
5. f(x) = is not defined for-
x 2  3x  2
(A) x = 2 (B) x = 1, 2 (C) x = 2,–1 (D) x = 0
6. The domain of the function f(x) = x! is -
(A) (0,  ) (B) N (C) W (D) R+
7. The domain of function f(x) = log (3x –1) + 2 log (x +1) is -

1 
(A) [1/3,  ) (B) [–1,1/3] (C) (–1,1/3) (D)  ,  
3 
Page 33 FUNCTION

1 1
8. If f(x) = and g (x) = , then common domain of function is -
x 1 x 1
(A) {x | x <1, x  R } (B) {x | x  0, x  1, x  R}
(C) {1} (D) {–1}

9. The natural domain of the real valued function defined by f (x) = x 2  1 + x 2  1 is-
(A) 1 < x <  (B) (–  ,  ) – {–1, 1}
(C) –  < x <–1 (D) (–  ,  ) – (–1,1)
10. The domain of the function log 2
log 3
log 4(x) is -
(A) (1,  ) (B) (2,  ) (C) (3,  ) (D) (4,  )

2 1
11. If f(x) = 4  x  sin x  sin x , then the domain of f(x) is

(A) [-2, 0] (B) (0, 2] (C) [-2, 2) (D) [-2, 0)


12. The entire graphs of the equation y = x2 + kx – x +9 is strictly above the x-axis if and
only if
(A) k < 7 (B) –5 < k < 7 (C) k > –5 (D) –7 < k < 5
Range:

13. The range of f(x) = sin [x] is -
2
(A) {–1,1} (B) {–1,0,1} (C) {0,1} (D) [–1,1]

1
14. For real values of x, range of function y = is -
2  sin3x

1 1 1 1
(A) y 1 (B) – y 1 (C) – >y>–1 (D) >y>1
3  3  3 3
15. The range of f(x) = cos 2x – sin 2x contains the set -

(A) [2,4] (B) [–1,1] (C) [–2,2] (D) [ 2, 2]


16. Range of the function f(x) = sin2(x4) + cos2(x4) is-
(A) (–  ,  ) (B) {1} (C) (–1,1) (D) (0,1)
2 2 2
17. If a + b + c = 1, then range of ab + bc + ca is-
(A) [–1/2,  ) (B) (0,  ) (C) [–1/2,1] (D) [1,  )
18. The range of 5 cos x – 12 sin x + 7 is -
(A) [–6,20] (B) [–3,18] (C) [–6,15] (D) None of these

19. The range of the function y  2  x  1  x is

(A)  3, 6  (B)  6,5  (C) 0, 3  (D)  6,3 

20. The range of the function y = 2cos 2 x  3 cos x  1 is

 1   1 
(A) (0, 2) (B)  , 1 (C)  0,  (D) [0, 2]
2 2   2 2
Page 34 FUNCTION

 [x 2  x] 
21. Range of f(x) = tan   is (where [ ] denotes the greatest integer function)
 1  sin(cos x) 
(A) ( –  ,  ) ~ [0, tan 1] (B) (–  ,  ) ~ [tan 2, 0)
(C) [tan 2, tan 1] (D) (–  ,  )

  x2  e  
22. If  = sin  ln  2
–1   , then range of  is
  x 1  
(A) (0,  /2) (B) [0,  /2) (C) (0,  /2] (D) (-  /2,  /2)
23. If log3 (x2 – 6x + 11) < 1, then the exhaustive range of values of x is
(A) (–  , 2)  (4,  ) (B) (2, 4)

(C) (–  , 1)  (1, 3)  (4,  ) (D)  , 1  1, 3


Even and odd Function:

sin4 x  cos 4 x
24. The function f(x) = is -
x  tan x
(A) odd (B) Even
(C) neither even nor odd (D) odd and periodic
25. A function is called even function if its graph is symmetrical w.r.t.-
(A) origin (B) x = 0 (C) y = 0 (D) line y = x
26. A function is called odd function if its graph is symmetrical w.r.t.-
(A) Origin (B) x = 0 (C) y = 0 (D) line y = x
27. The even function is-
(A) f(x) = x2 (x2 +1) (B) f(x) = sin3 x + 2
(C) f(x) = x (x +1) (D) f(x) = tan x + c
28. A function whose graph is symmetrical about the y-axis is given by-

(A) f(x) = loge (x + x2  1 ) (B) f(x + y) = f(x) + f(y) for all x, y  R


(C) f(x) = cos x + sin x (D) None of these
29. In the following, odd function is -
(A) cos x2 (B) (ex + 1)/(ex – 1)
(C) x2 – |x| (D) None of these
2
30. The function f(x) = x – |x| is -
(A) an odd function (B) a rational function
(C) an even function (D) None of these
Periodic Function:
31. The period of sin4 x + cos4 x is -
(A)  (B)  /2 (C) 2  (D) None of these
32. The period of function |cos 2x| is -
(A)  (B)  /2 (C) 4  (D) 2 
33. The period of the function f(x) = log cos 2x + tan 4x is -
(A)  /2 (B)  (C) 2  (D) 2  /5
Page 35 FUNCTION

1
34. The period of the function f(x) = 2 cos (x–  ) is -
3
(A) 6  (B) 4  (C) 2  (D) 
35. In the following which function is not periodic-
(A) tan 4x (B) cos 2  x (C) cos x 2 (D) cos2x
36. The period of function f (x) = |sin3 (x/2)| is
(A) 4  (B) 16  (C) 2  (D) None of these
37. If f(x) + f(x + a) + f(x + 2a) + … + f(x + na) = constant;  x  R and a > 0 and f(x) is
periodic, then period of f(x), is
(A) (n + 1)a (B) e(n+1)a (C) na (D) ena
100
38. If f(x) + f(x + 4) = f(x + 2) + f(x + 6); x  R , and f(5) = 10, then  f(5  8r) equal to
r 1

(A) 1000 (B) 100 (C) 10000 (D) 10


39. Which of the following functions is periodic ?
(A) f(x) = x – [x] (where [x] is g.i.f.)
1
(B) f(x) = sin for x  0 , f(0) = 0
x
(C) f(x) = x cos x

(D) f(x) = sin x


Mapping:
40. If f : I  I,f (x) = x3+ 1, then f is -
(A) one - one but not onto (B) onto but not one-one
(C) One-one onto (D) None of these
41. If f : R  R, f(x) = ex + e–x, then f is -
(A) one-one but not onto (B) onto but not one-one
(C) neither one-one nor onto (D) both one-one and onto
42. If f : R  R , f(x) = sin2 x + cos2 x , then f is -
(A) one-one but not onto (B) onto but not one-one
(C) neither one-one nor onto (D) both one-one onto
43. Which of the following functions from A = {x: –1  x  1} to itself are bijections ?

x  x 
(A) f(x) = (B) g(x) = sin   (C) h(x) = |x| (D) k(x) = x2
2  2 
44. Which of the following function defined from R to R is onto ?
(A) f(x) = |x| (B) f(x) = e–x (C) f(x) = x3 (D) f(x) = sin x.
45. If f : I  I, f(x) = x 2 – x, then f is -
(A) one-one onto (B) one-one into
(C) many-one onto (D) many-one into
46. If S be the set of all triangles and f : S  R+, f (  ) = Area of  , then f is -
(A) One-one onto (B) one-one into
(C) many-one onto (D) many-one into
Page 36 FUNCTION
47. Let f : R  R be a function defined by f(x) = x + x 2 , then f is-
(A) injective (B) surjective (C) bijective (D) None of these

+
x2  e
48. The function f : R  (1, e) defined by f(x) = 2 is
x 1
(A) one–one but not onto (B) onto but not one–one
(C) both one–one and onto (D) neither one–one nor onto

49. If f : D  [–2, 2] and f(x) = cosx – 3 sinx and f(x) is bijective function, then D may be

  2    4 
(A) [0,  ] (B)   , (C)  ,  (D) 0, 2
 3 3  3 3 

x2
50. If the function f : R  A given by f (x) = is a surjection, then A is
x2 1
(A) R (B) [0, 1] (C) (0, 1] (D) [0, 1)

x
51. If f : [0, )  [0,  ) and f(x) = then f is
1 x
(A) one - one and into (B) onto but not one - one
(C) one - one and onto (D) many - one into
52. Let R be the set of real numbers. If f : R  R is a function defined by f(x) = x2 then f is
(A) Injective but not surjective (B) Surjective but not injective
(C) Bijective (D) neither injective nor surjective
Composite Function:
53. gof exists, when-
(A) domain of f = domain of g (B) co-domain of f = domain of g
(C) co-domain of g = domain of g (D) co-domain of g = co-domain of f
54. If f : R  R, f(x) = x2 – 5x + 4 and g : R  R, g(x) = log x , then the value of (gof) (2) is -
(A) 0 (B)  (C) –  (D) Undefined
55. If f : R  R, g : R  R and f(x) = 3x + 4 and (gof) (x) = 2x – 1, then the value of g(x) is -

1
(A) 2x – 1 (B) 2x – 11 (C) (2x – 11) (D) None of these
3
56. If f(x) = ax + b and g(x) = cx + d, then f(g(x)) = g(f(x)) is equivalent to-
(A) f(a) = g(c) (B) f(b) = g(b) (C) f(d) = g(b) (D) f(c) = g(a)

1 x
57. If f(x) = , then f [f (sin  )] equals -
1 x
(A) sin  (B) tan (  /2) (C) cot (  /2) (D) cosec 
58. If f(x) = (a – x n)1/n, n  N, then f [f(x)] is equal to-
(A) 0 (B) x (C) xn (D) (an – x)n
59. If f(x) = log x, g(x) = x3, then f[g(a)] + f [g(b)] is equal to-
(A) f [g(a) + g(b)] (B) 3 f(ab) (C) g [f(ab)] (D) g [f(a) + f(b)]
Page 37 FUNCTION
60. If f (x) = x3 – x and g(x) = sin 2x, then -
(A) g [f(1)] = 1 (B) f (g (  /12)) = – 3/8
(C) g {f(2)} = sin 2 (D) None of these

x
61. If f(x) = 3x, g(x) = , h(x) = f(g(x)) then h(h(..... n times)) equal to
3
(A) xn (B) x (C) 3nx (D) 2x

1  x  3x  x 3
62. Given f(x) = log   and g(x) = then fog (x) equals
1 x  1  3x 2
(A) – f(x) (B) 3f (x) (C) [f(x)]3 (D) 2f(x)

2
63. If g( f(x)) = sin x and f (g(x))  sin x   , then

(A) f(x) = sin 2 x , g( x )  x (B) f(x) = sin x, g(x) = x

(C) f(x) = x 2 , g( x )  sin x (D) f and g cannot be determined

Inverse Function:
64. If f : R  R, f(x) = x2 + 3, then pre- image of 2 under f is -

(A) {1,–1} (B) {1} (C) {–1} (D) 


65. Which of the following functions has its inverse-
(A) f : R  R , f(x) = ax (B) f : R  R, f(x) = |x| + |x – 1|
(C) f : R0  R+, f(x) = |x| (D) f : [  , 2  ]  [–1,1], f(x) = cos x
66. The inverse of the function y = logex is -
(A) 10x (B) 10–x (C) ex (D) e–x
–1
67. If f : R  R f(x) = cos (5x + 2) then the value of f (x) is -

cos 1 (x)  2 cos 1 (x)


(A) (B) cos 1 ( x )  2 (C) 2 (D) Does not exist
5 5
68. The value of the parameter  , for which the function f(x) = 1 +  x,   0 is the
inverse of itself, is
(A) – 2 (B) – 1 (C) 1 (D) 2
69. Let f : N  Y be a function defined as f(x) = 4x + 3 where Y = {y  N : y = 4x + 3 for
some x  N}. its inverse is

y 3 y 3 3y  4 1
(A) g(y) = (B) g(y) = (C) g(y) = (D) g(y) = 4y  3
4 4 4

70. Let f:[-  /3, 2  /3]  [0,4] be a function defined as f(x)= 3 sin x – cosx + 2. Then
-1
f (x) is given by

1  x  2   1  x  2   2 1  x  2 
(A) sin   (B) sin   (C) – cos   (D) None of these.
 2  6  2  6 3  2 
Page 38 FUNCTION

71. Let f(x) = x  12  1, x  1 . Then the set S = {x : f(x) = f -1 (x)} is, if f is onto

 3  i 3 3  i 3 
(A) 0,  1, 2
,
2
 (B) {0, 1, -1}
 

(C) {0, -1} (D) 

72. If the function f: [2,  )  [1,  ) is defined by f (x) = 3x(x –2), then f-1 (x) is

(A) 1 + 1  log 3 x (B) 1 - 1  log 3 x (C) 1 + 1  log 3 x (D) does not exist

73. If f(x) = sinx + cosx, g(x) = x2 – 1, then g(f(x)) in invertible in the Domain

       
(A) 0,  (B)   ,  (C)   ,  (D)  0, 
 2  4 4  2 2
74. If f : R  R, f(x) = x3 + 3, and g : R  R, g(x) = 2x + 1, then f–1og–1(23) equals-
(A) 2 (B) 3 (C) (14)1/3 (D) (15)1/3
Value of Function:

2x
75. If f(x) = , then f (tan  ) equals-
1  x2

(A) cot 2  (B) tan 2  (C) sec 2  (D) cos 2 


76. If f(x) = log x, then correct statement is-
(A) f(x + y) = f( x ) + f(y) (B) f(x + y) = f( x) . f(y)
(C) f(xy) = f(x) + f(y) (D) f(xy) = f( x) . f(y)
77. If f(x) = 2 cos x + sin2 x, then f(2  – x) equals-
(A) – f(x) (B) f(x) (C) – 2f(x) (D) 2f(x)
78. If f : R  R , f(x) = 2x ; g : R  R, g(x) = x + 1, then (f .g) (2) equals -
(A) 12 (B) 6 (C) 3 (D) None of these

b( x  a) a( x  b)
79. If f(x) = (b  a) + (a  b) , then f(a + b) =

(A) f(a). f(b) (B) f(a) – f(b) (C) f(a) /f(b) (D) f(a) + f(b)

f(xy)  f(x / y)
80. If f(x) = cos (log x), then f(x)f(y) equals

(A) 1 (B) –1 (C) 0 (D) 2


81. If f (x) = |x| + |x – 1|, then for 0 < x < 1, f (x) equals
(A) 1 (B) –1 (C) 2x + 1 (D) 2x – 1
82. If f(x) = (ax – c)/(cx – a) = y, then f(y) equals
(A) x (B) 1/x (C) 1 (D) 0
Page 39 FUNCTION

1
83. If f(x) = x + , then
x

(A) f(x2) = [f(x)]2 (B) f(x + y) = f(x) + f(y)


(C) f(–x) = f(x) (D) f(1/x) = f(x)
84. If f(x) = x2 – x–2, then f(1/x) equals

1
(A) f ( x) (B) –1/f(x) (C) f(x) (D) – f(x)

85. If f(x) + 2f(1 – x) = x2 + 1;  x  R then f(x) is

1 2 2 2 1 2 2 2
(A)
3

x  4x  3  (B)
3
(x + 4x – 3) (C)
3
(x – 4x + 3) (D)
3
(x – 4x + 3)

86. Let f be a function satisfying 2f(x) – 3f(1/x) = x2 for any x  0, then the value of f(2) is

7
(A) –2 (B) –7/4 (C)  (D) 4
8

1 1
87. If f(x) f   = f(x) + f   ; x  R – {0}, where f(x) be a polynomial function and f(5) =
x x
126 then f(3) =
(A) 28 (B) 26 (C) 27 (D) 25

MULTIPLE CORRECT TYPE QUESTIONS

 2   2 
1. If f(x) = cos   x  sin    x, [x] is G.I.F. then
2  2

 1
(A) f(0) = 1 (B) f  3   (C) f   / 2   0 (D) f ( ) = 0
  3 1

2. If the domain of f(x) be (–1, 2), then

1 2
(A) domain of f(sinx) will be (–  ,  ) (B) domain of f(log x) will be  ,e 
e 
(C) domain of f([x]) will be [0, 2) (D) none of these

3. Among the following which is / are true

(A) Range of x 2  2x  1 + x 2  4x  4 is [3, )

2 2
(B) Range of sin x     sin x  (where  is g.i.f.) is {0, –1})

(C) The period of sin2 x  cos2 x   sin x   


2
cos2 x 
Page 40 FUNCTION


(where 
 denotes decimal part) is
2

       1
(D) If f(x) = sin x sin   x  sin   x  then f   =
 3   3   18  8

4. 
2
Let f(x) = loge x  x  1 then 
 2
(A) f 2x 1  x  2f(x) 
(B) f 3x  4x 3  3f(x) 
e f (x ) – e f (x )
 2 2

(C) f x 1  y  y 1  x  f(x)  f(y) (D)
2
x

5. Let f(x) = x2 and g(x) = x , (where x > 0), then

(A) f  g(x)  x (B) g  f(x)  x

1 1
(C) The least value of f  g(x)  is 2 (D) The least value of g  f(x)  f g(x) is 2
g  f(x)  
6. Let f : A  B be a function defined by f(x) = 1  x2
(A) f(x) is one-one if A = [0, 1] (B) f(x) is onto if B = [0, 1]
(C) f(x) is one-one if A = [–1, 0] (D) f(x) is into if B = [–1, 1]
–1
7. If f(x) = cos (cosx) , then
(A) f(x) = x, 0  x   (B) f(x) = 2  – x,   x  2
(C) f(x) = 2  + x , 2   x  3  (D) f(x) = –2  – x, 3   x  4 
8. Total number of solutions of the equation 2x|2 – |x|| = 1 are
(A) 2 (B) 1 (C) 0 (D) 3
9. Let f(x) = cos x + sin x
(A) Domain of f(x) is R (B) Range of f(x) is [–2, 2]

(C) Range of f(x) is   2, 2  (D) Period of f(x) is 2


10. Among the following which is / are true
(A) If f, g are only two functions then fog = gof always
(B) f(f(x)) is always defined, where ‘f’ is any function
(C) f(g(x)) = x  g = f–1 (x)
(D) none

1
11. If 1 + (x2 – 1) + (x2 – 1)2 + (x2 – 1)3 + .... up to  terms = then x belongs to
2  x2


(A) 0, 2  
(B)  2, 2  
(C)  2, 2 – {0}  (D) none of these

12. The value of f(x) = 3 sinx – 4 cosx + 5 lies in the interval


(A) [0, 10] (B) (0, 10) (C) [–5, 10] (D) none of these
Page 41 FUNCTION

13. Let f :[0, 2]  [0, ) defined as f(x) =  x 2  4 , then the values of ‘x’ for which f(x) = f 1  x  is

(A)  2 (B) 2 (C) 0, 2 (D) none

14. Let g(x) be a function on [-1, 1]. If the area of the equilateral triangle with two of its

3
vertices at (0, 0) and [x, g(x)] is , then the function g(x) is
4

(A) g(x) =  1 x 


2
(B) g ( x )  1  x 
2

(C) g(x) =  1 x 


2
(D) g( x )  1  x 
2

   
15. If f(x)  cos2 x  cos2   x   cos x.cos   x  then
3  3 

 
(A) f(x) is an even function (B) f  f 
8  4
(C) f(x) is a constant function (D) f(x) is not a periodic function

16. Let f(x) = 9  x 2  x 2  9 , then

(A) domain of f(x) is 3 (B) f(x) is many one function


(C) Range of f(x) is {0} (D) ‘f’ is an even function

17. 2
 2
 
f  x   sgn x , g  x   x x  1 , h  x   x  1 sin x then which of the following is/are periodic

functions

(A) g  f  x  (B) f  g  x  (C) h  f  x   (D) g h  x  

COMPREHENSION TYPE QUESTIONS


Passage I
A function f : A  B is said to be injective if distinct elements in A have distinct
images in B. And surjective if f(A) = B. Then answer the following

1  cos 2x
1. If the function f : A  B defined by f  x   is injective then the set A can be
2

(A)  0,  (B)  ,  (C)   / 2, 0 (D)  , 0

2. If the function f : R  B defined by f  x   x 2 is surjective then the set ‘B’ can be

(A) ( , 0] (B) [0,  ) (C) (0,  ) (D) R


3. The function f : R  B defined by f(x) = [x] + [–x] (where [.] is g.i.f) is surjective then B =
(A) R (B) [0, 1] (C) [–1, 0] (D) {–1, 0}
Page 42 FUNCTION
Passage II
The accompanying figure shows the graph of a function f(x) with domain [-3, 4] and
range [-1, 2]

Y
2

I. –3 –2 –1 0 1 2 3 4
X II.
–1

On the basic of above information, answer the following questions:


4. Figure (ii) represents the graph of the function
(A) f(x) (B) f(|x|) (C) |f(x)| (D) |f(|x|)|
5. The domain and range respectively of
(A) f(-x) are [-4, 3] and [-2, 1]
(B) f(x) – 1 are [-3, 4] and [-1, 2]
(C) g(x) + 2 are [-3, 4] and [-2, 4]
(D) –f(x + 1) are [-4, 3] and [-2, 1]
6. The number of solution of figure (ii) and (2x – 6)2 + 4y2 = 49 are
(A) 2 (B) 4 (C) 6 (D) 1
Passage III
If f(x) = 0 ; if x  Q
= 1 ; if x  Q.
then answer the following questions-
7. f(x) is -
(A) an even function (B) an odd function
(C) Neither even nor odd function (D) one-one function
8. f (f(x)) is-
(A) a constant function (B) an even function
(C) an odd function (D) many one function
9. Domain of g(x) = ln (sgn f(x)) is-
(A) R (B) set of all rational numbers
(C) set of all irrational number (D) R+

Passage IV
Consider the function

 1
x  [ x ]  ; if x 
f(x) =  2
 0 ; if x 

where [.] denotes greatest integer function.


If g(x) = max. {x2, f(x), | x |} ; –2  x  2, then.
Page 43 FUNCTION
10. Range of f(x) is-

 1 1  1 1  1 1
(A) [0, 1) (B)  2 , 2  (C)   ,  (D)  , 
   2 2  2 2
11. f(x) is-
(A) non periodic (B) periodic with period 1
(C) periodic with period 2 (D) periodic with period 1/2
12. The set of values of a, if g(x) = a has three real and distinct solutions, is -
(A) (0, 1/2) (B) (0, 1/4) (C) (1/4, 1/2) (D) (0, 1)

MATRIX-MATCH QUESTIONS
1. The domain of the function
Column-I Column-II

 3  2x 
(A) 3  x  sin1   (p) kI(2k,(2k  1))
 5 

(B) log10 (1  log10 (x 2  5x  16)) (q) [  4,   ]  [0,  ]

 2 
(C) cos 1   (r) (2, 3)
 2  sin x 
(D) sin x  16  x 2 (s) [–1, 3]

2. Column-I Column-II
x
(A) The period of the function sin log   (p) 1
3
(where {x} denotes the fractional part of ‘x’) is
(B) The total number of solutions of the equation (q) 2
log e x  sin x in   / 2,  / 2 is
2 2
(C) The least value of the function f  x   sec x  cos ec x is (r) 4
(D) Let f(x) = | sin x | + | cos x | then number of elements (s) 3
in range set of [f(x)] where ([  ] is G.I.F is)
(t) infinite
3. Match the following:
Column-I Column-II
(A) Let f(x) = max {1 + sin x, 1, 1 – cos x}, x  [0, 2  ] (p) g(f(1)) = 1
and g(x) = max {1, |x – 1|}, x  R, then

 1+ x   3x  x 3 
ln
(B) Let f(x) =   ; x  (-1,1) and g(x)   2  , then (q) f(g(0)) = 0
 1- x   1  3x 
(C) Let f(x) = 1 + x 2 and g(x) = x – x2, then (r) f(g(0)) = 1
(s) g(f(0)) = 1

  e 1
(t) g  f    1
  e 1
Page 44 FUNCTION

INTEGER TYPE QUESTIONS

1
1. If f(x) = (1005 – x10)1/10, then find the value of f(f(1024)).
210


r
2. An odd function is symmetric about the vertical line x = a (a > 0) and if [f (1 4ar)]
r0
= 8,

then find the value of 8f(1).


3. Let f : R  R such that f(x – f(y)) = f(f(y)) + x f(y) + f(x) – 1;  x, y  R. Then find the
value of |(f(16))| - 125.
4. Let f be a function from the set of positive integers to the set of real number such
that
n

(i) f(1) = 1 (ii)  r f (r)


r 1
= n (n + 1) f(n);  n  2 then find the value of 2126 f(1063).

5. Let f(x) be a function such that f(x – 1) + f(x + 1) = 2 f(x);  x  R. If f(2) = 7 then find
17

the value of  f(2  8r)  7 .


r 0

6. If f(x), g(x) are any two real valued functions such that |f(x) + g(x)|  |f(x)| + |g(x)|
100
and g(x)  0, f(x).g(x)  0 then find the value of  f(r) .
r=1

x 4 + x 2 +1 n
7. If f(x) = 2 , then find the value of f(  ) where ‘  ’ is the non real root of the
x - x +1
equation z3 = 1 and ‘n’ is a multiple of ‘3’.

PREVIOUS YEARS QUESTIONS (AIEEE/ JEE MAINS)

1. Which of the following is not a periodic function - [AIEEE 2002]

(A) sin 2x + cos x (B) cos x (C) tan 4x (D) log cos 2x
2. The period of sin2 x is- [AIEEE 2002]
(A)  /2 (B)  (C) 3  /2 (D) 2 
3. The function f : R  R defined by f(x) = sin x is- [AIEEE-2002]
(A) into (B) onto (C) one-one (D) many-one

2x
4. The range of the function f(x) = , x  2 is - [AIEEE-2002]
2x
(A) R (B) R – {–1} (C) R – {1} (D) R – {2}

5. The function f(x) = log (x + x 2  1 ), is- [AIEEE 2003]


(A) neither an even nor an odd function (B) an even function
(C) an odd function (D) a periodic function
Page 45 FUNCTION

3
6. Domain of definition of the function f(x) = + log10 (x3 – x), is- [AIEEE 2003]
4  x2
(A) (– 1, 0)  (1, 2)  (2,  ) (B) (1, 2)
(C) ( – 1, 0)  (1, 2) (D) (1, 2)  (2,  )
n

7. If f : R  R satisfies f(x+ y) = f(x) + f(y), for all x, y  R and f(1) = 7, then  f(r) is-
r 1

[AIEEE 2003]

7n(n  1) 7n 7(n  1)
(A) (B) (C) (D) 7n (n+1)
2 2 2
8. A function f from the set of natural numbers to integers defined by

 n 1
 2 ,when n is odd
f(n)=  is [AIEEE 2003]
  n , when n is even
 2

(A) neither one-one nor onto (B) one-one but not onto
(C) onto but not one-one (D) one-one and onto both
7– x
9. The range of the function f(x) = Px–3 is- [AIEEE 2004]
(A) {1, 2, 3} (B) {1, 2, 3, 4, 5, 6}
(C) {1, 2,3,4} (D) {1, 2, 3, 4, 5}

10. If f : R  S, defined by f(x) = sin x – 3 cos x + 1 is onto, then the interval of S is-
[AIEEE 2004]
(A) [0, 3] (B) [–1, 1] (C) [0, 1] (D) [–1, 3]
11. The graph of the function y = f(x) is symmetrical about the line x = 2, then-
[AIEEE 2004]
(A) f(x+ 2) = f(x – 2) (B) f(2 + x) = f(2 – x)
(C) f(x) = f(–x) (D) f(x) = – f(–x)

sin1(x  3)
12. The domain of the function f(x) = is- [AIEEE 2004]
9  x2
(A) [2,3] (B) [2,3) (C) [1,2] (D) [1, 2)

2x
13. Let f : (–1, 1)  B, be a function defined by f(x) = tan–1 , then f is both one-one
1 x2
and onto when B is the interval - [AIEEE-2005]

         
(A)  0,  (B) 0,  (C)   ,  (D)   , 
 2  2  2 2  2 2
14. A real valued function f(x) satisfies the functional equation f(x – y) = f(x) f(y) – f (a – x) f(a + y)
where a is a given constant and f(0) = 1, then f(2a – x) is equal to - [AIEEE-2005]
(A) –f(x) (B) f(x) (C) f(a) + f(a – x) (D) f(–x)
Page 46 FUNCTION
   
15. The largest interval lying in  2 , 2  for which the function

 x2 x  
 f(x)  4  cos 1   1  log(cos x) is defined, is- [AIEEE 2007]
 2  
        
(A) [0,  ] (B)  2 , 2  (C)  4 , 2  (D) 0, 2 
 
16. Let f : N  Y be a function defined as f(x) = 4x + 3 where Y = {y  N : y = 4x + 3 for
some x  N}. Show that f is invertible and its inverse is [AIEEE 2008]

y3 y3 y3 3y  4


(A) g(y) = 4 + (B) g(y) = (C) g(y) = (D) g(y) =
4 4 4 3

17. For real x, let f(x) = x3 + 5x + 1, then - [AIEEE 2009]

(A) f is one – one but not onto R (B) f is onto R but not one – one
(C) f is one – one and onto R (D) f is neither one – one nor onto R

18. Let f(x) = (x + 1)2 –1, x > –1 [AIEEE 2009]


Statement – 1 : The set {x : f(x) = f–1(x)} = {0, –1}.
Statement – 2 : f is a bijection.
(A) Statement -1 is true, Statement -2 is true; Statement -2 is a correct explanation
for Statement -1
(B) Statement -1 is true, Statement -2 is true; Statement -2 is not a correct
explanation for Statement -1.
(C) Statement -1 is true, Statement -2 is false.
(D) Statement -1 is false, Statement -2 is true.

1
19. The domain of the function f(x) = is [AIEEE-2011]
| x | x

(A) (0,  ) (B) (-  , 0) (C) (-  ,  ) – {0} (D) (-  ,  )


20. If a  R and the equation –3(x – [x])2 + 2(x – [x]) + a2 = 0 where [x] denotes the greatest
integer  x) has no integral solution, then all possible values of a lie in the interval ?
[JEE MAINS-2014]
(A) (-1, 0)  (0, 1) (B) (1, 2)
(C) (–2, –1) (D) (,  2)  (2,  )

1
21. If f  x   2f    3x, x  0 , and S   x  R : f  x   f  –x  ; then S: [JEE MAINS-2016]
x
(a) is an empty set (b) contains exactly one element
(c) contains exactly two elements (d) contains more than two elements
Page 47 FUNCTION

PREVIOUS YEARS QUESTIONS (IIT JEE / JEE ADVANCED )

-1
1. If f(x) = 3x - 5, then f (x) [IIT - 1998]

1 x 5
(A) is given by (B) is given by
3x  5 3
(C) does not exist because f is not one-one (D) does not exist because f is not onto
2. The f : R  R be any function. Define g : R  R by g(x) = | f(x) | for all x. Then g is
[IIT - 2000]
(A) onto if f is onto (B) one–one if f is one-one
(C) continuous if f is continuous (D) differentiable if f is differentiable
3. The domain of definition of the function, f(x) given by the equation, 2x + 2y = 2 is
[IIT - 2000]
(A) 0  x  1 (B) 0  x  1

(C)    x  0 (D)    x  1

log 2  x  3 
4. The domain of definition of f(x) = is [IIT - 2001]
x 2  3x  2

(A) R-{-1, -2} (B)  2,   (C) R-{-1, -2, -3} (D)  3,    1, 2

x
5. Let f(x) = , x  1 . Then for what value of  is f( f(x)) = x [IIT - 2001]
x 1

(A) 2 (B)  2 (C) 1 (D) –1

 1, x  0
6. Let g(x) = 1 + x - [x] and f(x) = 0, x  0 . Then for all x, f(g(x)) is equal to [IIT - 2001]
1, x0

(A) x (B) 1 (C) f(x) (D) g(x)

1
7. If f : 1,    2,   is given by, f(x) = x  , then f -1(x) equals [IIT - 2001]
x

x  x2  4 x x  x2  4
(A) (B) (C) (D) 1  x 2  4
2 1  x2 2

8. Let function f : R  R be defined by f(x) = 2x + sin x for x  R , then f is [IIT - 2002]


(A) one-to-one and onto (B) one-to-one but NOT onto
(C) onto but NOT one-to-one (D) neither one-to-one nor onto
9. Suppose f(x) = (x + 1)2 for x  1 . If g(x) is the function whose graph is the reflection
of the graph of f(x) with respect to the line y = x, then g(x) equals [IIT - 2002]

1
(A)  x  1, x  0 (B) (x  1)2 , x  1
Page 48 FUNCTION

(C) x  1, x  1 (D) x  1, x  0

 1 
10. Domain of definition of the function f(x) =  sin  2x   6  is [IIT - 2003]
 

 1 1  1 1  1 1  1 1
(A)   ,  (B)   ,  (C)   ,  (D)   , 
 4 2  2 2  2 9  4 4

x2  x  2
11. Range of the function f(x) = ; x  R is [IIT - 2003]
x2  x  1

(A) (1,  ) (B) (1, 11/7] (C) (1, 7/3] (D) [1, 7/5]
12. If f(x) = x2 + 2bx + 2c2 and g(x) = – x2 – 2cx + b2 such that min f(x) > max g(x), then the
relation between b and c, is [IIT - 2003]

(A) no real value of b and c (B) 0  c  b 2

(C) | c |  | b | 2 (D) | c |  | b | 2

13. Let f(x) = sin x + cos x and g(x) = x2 – 1, then domain for which gof is invertible, is
[IIT- 2004]

    2       
(A)  0,  (B)  ,  (C)   ,  (D)   , 
 2 2 3   2 3  4 4
14. If X and Y are two non-empty sets where f : X  Y is function is defined such that

f  c   f  x  : x  C for C  X and f 1  D   x : f (x)  D for D  Y, for any A  Y and

B  Y then [IIT- 2004]

(A) f 1 (f (A))  A (B) f 1 (f (A))  A only if f  X   Y

(C) f (f 1 (B))  B onlyif B  f (x) (D) f (f 1 (B))  B

 x, if x is rational 0, if x is rational


15. f(x) =  , and g(x) =  , then f – g is [IIT-2005]
0, if x is irrational  x, if x is irrational
(A) one–one and into (B) neither one–one nor onto
(C) many one and onto (D) one–one and onto

bx
16. Let f : (0, 1)  R be defined by f(x) = , where b is a constant such that 0 < b < 1.
1  bx
Then [IIT-2011]

1
(A) f is not invertible on (0, 1) (B) f  f 1 on (0, 1) and f (b) 
f (0)

1
(C) f  f 1 on (0, 1) and f (b)  (D) f 1 is differentiable on (0, 1)
f (0)
Page 49 FUNCTION

17. The function f : [0, 3]  [1, 29], defined by f(x) = 2x3 – 15x2 + 36x + 1, is [IIT-2012]
(A) one–one and onto (B) onto but not one–one
(C) one–one but not onto (D) neither one–one nor onto

2     
18. Let f : (–1, 1)  IR be such that f (cos 4)  2
for    0,    ,  . Then the
2  sec   4  4 2
1
value(s) of f   is are [IIT-2012]
3

3 3 2 2
(A) 1 (B) 1 (C) 1 (D) 1
2 2 3 3

  
19. Let f :   ,   R be given by f(x) = (log(sec x + tan x))3. Then [IIT 2014]
 2 2
(A) f(x) is an odd function (B) f(x) is a one-one function
(C) f(x) is an onto function (D) f(x) is an even function

   
20. Let f(x) = sin  sin  sin x   for all x  R and g(x) = sin x for all x  R . Let (f o g)(x)
 6  2  2
denote f(g(x)) and (g o f)(x) denote g(f(x)). Then which of the following is (are) true ?

 1 1  1 1
(A) Range of f is   ,  (B) Range of f o g is   ,  [IIT 2015]
 2 2  2 2

f(x) 
(C) lim
x 0 g(x)

6 (D) There is an x  R such that (g o f)(x) = 1

21. Let f : R  R, g : R  R and h : R  R be differentiable functions such that f(x) = x3


+ 3x + 2, g(f(x)) = x and h(g(g(x)) = x for all x  R . Then [JEE ADV. 2016]

1
(a) g '  2  (b) h' 1  666 (c) h(0) = 16 (d) h(g(3)) = 36
15

SUBJECTIVE TYPE QUESTIONS


1. A function f : R  R , where R is the set of real numbers, is defined by,

 x 2  6x  8
f(x)= . Find the interval of values of  for which f is onto. Is the function
  6x  8x 2
one-to-one for  =3 ? Justify your answer. [IIT - 1996]
2. Let f : {x, y, z}  {a,b,c} be a one-one function. It is known that only one of the following
statements is true : [IIT - 1996]
(i) f ( x )  b (ii) f(y) = b; (iii) f ( z )  a . Find the function f.
3. Let f(x) = Ax2 + Bx + C where A, B, C are real numbers. Prove that if f(x) is an integer
whenever x is an integer, then the numbers 2A, A + B and C are all integers. Conversely,
prove that if the numbers 2A, A + B and C are all integers then f(x) is an integer
whenever x is an integer. [IIT - 1998]
Page 50 FUNCTION

x 2  6x  5
4 Let f(x) =
x 2  5x  6
Column I Column II
(A) If -1 < x < 1, then f(x) satisfied (p) 0 < f(x) < 1
(B) If 1 < x < 2, then f(x) satisfies (q) f(x) < 0
(C) If 3 < x < 5, then f(x) satisfies (r) f(x) > 0
(D) If x > 5, then f(x) satisfies (s) f(x) < 1 [IIT - 2007]
5. If the function f(x) = x3 + ex/2 and g(x) = f–1(x), then the value of g (1) is [IIT - 2009]

ANSWER KEY

DRILL - I
 5 12   1 
1.  ,  2. ( , 4) 3. (, 7)    ,3 
3 7  2 
4. (3, 4) 5. {-1} 6. (1, 2)  (2, 3)
7. (–  ,–2)  (–2, –1)  (1,  ) 8. (5/2, 8)
9. (–3, 1) 10. (–  ,  ) 11. (–1, 1)  (4, 6)
12. (–1,  ) 13. [  2 , –1)  (–1, 2 ]  [3, 4)
14. (–  , –5)  (1, 2)  (6,  ) 15. (1, 2)  (7,  )
16. (– 2 , 0)  (1, 2 )  (2,  ) 17. ( , 2]  {0,3}
18. {2,3}
19. (a) x  (, 6)  (1,3)  (7, ) (b) x  ( , 6]  [1,3]  (7, )
(c) x  (6,1)  {2,0}  (3,7) (d) x  [6,0)  (0,1]  [3,7)

DRILL - II
1. A. {4/3} B. {-9/2, 13/4} C. All x  [2,  ).
D. x = {0, 1} E. (–  , 0]  [4,  ) F. {0}  (1,  )
2. {-1}  [1,  )
3. A. (2,3) B. (-  , 1/6 ]  [ 3/2 ,  ) C. [ -1,  )
D. ( -  , -2]  [2,  ) E. (-1 , 5)
F. (-1 , 2 )  (3 , 6 ) G. (1, 3)
H. (2 , 5)
4. [-2  , -  ]  [-1, 0]  [1,  ]  {2  }
DRILL - III
1. 75 2. 15 3. 1  x < 9 4. (-  , 1]  [2,  )
8 19 29 97
5. x = 0, 5/3 6. x = 1, 7. x 8. , ,
3 6 12 24
9. no real solution
Page 51 FUNCTION

DRILL - IV
1. ( 2, ) 2. [1, 6] 3. [ 1  3, 1  3]
4. R – Z 5. (0, 1)  (1,  ) 6. (, 2)  (2, )
7. [ 2,1)  {0} 8. (3, ) 9. R – {1, 2}
10. (-  , –1)  (4,  ) 11. (–1, 0)  (1, 2)  (2,  ) 12. (0, 1]  [4, 5)
13. x < 1 14. [0, 1] 15. (, 2]  (0,1)

 1 
16. x   ,    0,1,2 17. x  (8, 10) 18. x  (2, 3)
2 
19. (–1, 1) –{0}  (2 ,  ) 20. [3,  )
 –2 
21. (0, 16) 22. (0, 125] 23.  3, 
 3 
24. (5,  ) 25. x < 0 26. 
27. (–  , 2)  [3,  ) 28. (–2, –1]  [1, 2)
29. R – {(0, 1)  {1, 2, 3, 4, 5, 6, 7}  (7, 8)}
30. x  (–  , –7]  [7,  ) 31. (1/2, 1)
 n 
32. R – [1, 2) 33. (-2, 1) 34. 2n  + 35. n   1
2 2
DRILL - V
1. {–1, 1} 2. [2, 16] 3. [5, 6) 4. [–2,  ) 5. {1, 3}
 1 1
6.   ,  7. (0, 1] 8. [1,  ) 9. [4,  ) 10. (-  , 4]
 2 2

3  3  1 
11.  ,   12.  ,1 13. {0,-1} 14.  , 1 15. [0, 1/2]
4  4  3 

3 
16.  ,   17. (1,  ) 18. {0, 1} 19. {1} 20. [-5, 5]
4 

 11 
21. [5- 25  12 3 , 5+ 25  12 3 ] 22. (-1, 1) 23. log e ,
 3 

1  3 
24. [0, 1/2) 25.  , 1 26. [2, 2 2 ] 27. [1, 2] 28.  ,3
2  2 
29. {0} 30. Domain [–1, 1] Range [0,2] 31. {1} 32. {0}

DRILL - VI
1. (a) not equal function (b) equal function (c) not equal function
(d) not equal function (e) equal function (f) not equal function
(g) equal function (h) equal function (i) not equal function
(j) equal function
2. (a) x  (0, )  {1} (b) equal function
Page 52 FUNCTION
DRILL - VII
1. Determine the nature of the following functions for even and odd.
(a) neither even nor odd (b) even (c) odd (d) even
(e) odd (f) odd (g) odd (h)neither odd nor even
2. Find out whether the following functions is even or odd.
(a) even (b) odd (c) even (d) even
 a 2x  1   a 2x  1 
5. (a) f ( x )   
x   x 

 2a   2a 

 (1  x )100  (1  x )100   (1  x )100  (1  x )100 


(b) f ( x )      
 2   2 

 x
(c) f (x )   cos    sin2x  tan x 
 2 

a (e 2x  1) a (e2x  1)
(d) f (x )  
2e x 2e x

 e x  sin x ; 4  x  0
6. f 0 (x )    x
 e  sin x ; 0  x  4

 e x  sin x ; 4  x  0
f e (x )    x
e  sin x ; 0  x  4

 x 2  x  sin x  cos x  ln(1 | x |); 0  x  1


7. f 0 (x )   2
 x  x  sin x  cos x  ln(1 | x|); 1  x  0

 x 2  x  sin x  cos x  ln(1 | x |); 0  x  1


f e (x )   2
 x  x  sin x  cos x  ln(1 | x|); 1  x  0

DRILL - VIII
1. (a)  (b) 2  (c) 2  (d) no (e) 1
2. Find the period of
(a)  /2 (b) 4  (c) does not exist (d)  /2

(e) 2  (f) 2  (g) (h) 2 (i) 24
2
(j) does not exist
3. (a) 1 (b) 2n 
1
4. (a) n = ±1, ±3, ±5, ±15. (b)
3

DRILL - IX
1. one-one onto 2. many-one into 3. many-one into
4. one-one onto 5. many-one into 6. one-one onto
7. (a) one-one onto (b) one -one onto (c) many - one into
8. b2 < 3a {c - |d|} 9. a  (1,4) 10. n!
Page 53 FUNCTION

11. nn 12. 2n - 2 13.   [2, 14]


     a  2b 
16. x =    ,    ;  = tan-1   , Y = [c-r, c+r] where r  a 2  b2  2ab
a 
 2 2   
17. 3n -3(2n-1)

DRILL - X
x2 2
1. 2
2, x 2 2. gof = {(1,3), (3,1), (4,3)}, fog(x)= {(2,5), (5,2), (1,5)}
( x  1) x2 1
x
3. {-1, 1} 4. x3 5.
1  3x 2

2  x 0  x  1

6. (a)  2 - x 1  x  2 , (b) (gof)x = x ;  x  R
4 - x 2  x  3

7. 1 ;  x  R.

 3 x  1   x2  x  2 x  -1
2 x  1  1  x  0  2
x - x - 2 -1  x  0
8. (a)  , (b)  2
2 x  4 0  x 1 x  4 x  3 0  x 1
 4 x 1   x  2 x  3
2
x 1

 x  1 ; 2  x  1
  1  x ; 1  x  1
9. f  g  x      x  1 ; 1  x  0 , g  f  x    
 x 1 ; 3  x ; 1  x  3
 0x 2

1  x ; 0  x  1
3  x ; 1  x  2

10. g(f (x)   11. (i) a = – 1, b  R , (ii) a = 1, b = 0
 1  x; 2  x  3
5  x ; 3  x  4

 1 1  x  2, x  3

12. gof   0 x  1,2,3
 –1 x  1, 2  x  3

DRILL - XI

x  x2  4
1. f 1 (x)  2. f 1 (x)  2  4  x
2

x 5 x  x2  4 1 x
3. (a) (b) (c) (x-3)1/3 (d) y = (a  a  x )
3 2 2
(e) 3 sin x
Page 54 FUNCTION

4. f-1(x)=log(x+ x 2  1 ) 5. x = 1 6. (a) No. (b) No.


7. g ' (x)=1+{g(x)}3

DRILL - XII
1 a  5
3. x 4. f(x) = 2 2 
 bx   5. 217
a b  x  ab
2
3 e x   1 5m(m  1) an (n  1)
6. 7. 10 8. 9.
8 ex 2 2

x  15 x 1
10. 997.5 13. n + 15 15. f (x ) 
3 x 1
1
17. P(x) = x + 3, Q(x) = -x-2 18. 19. 1904, 1994
1994  2

SINGLE CORRECT TYPE QUESTIONS


1. C 2. A 3. C 4. B 5. B 6. C 7. D 8. B 9. D 10. D
11. D 12. B 13. B 14. A 15. D 16. B 17. C 18. A 19. A 20. C
21. D 22. C 23. B 24. A 25. B 26. A 27. A 28. D 29. B 30. C
31. B 32.B 33. B 34. A 35. C 36. C 37. A 38. A 39. A 40. A
41. C 42. C 43. B 44. C 45. D 46. C 47. D 48. C 49. B 50. D
51. A 52. D 53. B 54. D 55. C 56. C 57. A 58. B 59. B 60. B
61. B 62. B 63. A 64. D 65. D 66. C 67. D 68. B 69. B 70. C
71. C 72. A 73. B 74. A 75. B 76. C 77. B 78. A 79. D 80. D
81. A 82. A 83. D 84. D 85. C 86. B 87. A

MUTIPLE CORRECT TYPE QUESTIONS


1.A, B, C 2.B , C 3. A,B,C,D 4. A,B,C,D 5. A,B,C,D
6. A,B,C,D 7. A, B 8. D 9. A,C,D 10. D
11. A,B,C 12. A,C 13. B,C 14. B, C 15. A, B,C
16. A,B,C,D 17. A, C

COMPREHENSION TYPE QUESTIONS


1. C 2. B 3. D 4. D 5. D 6. D 7. A 8. A,B,C,D
9. C 10. C 11.B 12.C

MATRIX MATCH QUESTIONS


1. (A-s), (B-r), (C-p), (D-q) 2. A  S;B  Q;C  R ;D  P
3. A  P,S,T; B  Q,T  C  R

INTEGER TYPE QUESTIONS


1. 1 2. 7 3. 2 4. 1 5. 0 6. 0 7. 3
Page 55 FUNCTION

PREVIOUS YEAR QUESTIONS (AIEEE)


1. B 2. B 3. AD 4. B 5. C 6. A 7.A 8. D 9. A 10. D
11. B 12. B 13. D 14. A 15. D 16. C 17. C 18. C 19. B 20. A
21. C

PREVIOUS YEAR QUESTIONS (IIT JEE)


1. B 2. C 3. D 4. D 5. D 6. B 7. A 8. A 9.D 10.A
11. C 12. D 13. D 14. C 15. D 16. A 17. B 18. AB 19. ABC
20. ABC 21. (B, C)

SUBJECTIVE QUESTIONS:

1. 9
   , as domain is R      a can not be 3
8

Note : If f : R  {x : 8x 2  6 x    0, and x  R}  R , then   [ 2, 14] for onto function,


for   3 , f is not one to one

2. x, b, y, a , z, c


4. (A)  (p), (s); (B)  (q); (C)  (q); (D)  (p), (r)
5. (2)
Page 56

INVERSE TRIGONOMETRIC
FUNCTION
2
UNIT

 Theory

 Drill Exercises

 Solved Examples

 Exercises

 Single Correct Type Questions

 Multiple Correct Type Questions

 Comprehension Type Questions

 Matrix Match Questions.

 Integer Type Questions

 Previous Years Questions

 Answer Key
Page 57 INVERSE TRIGONOMETRIC FUNCTION
Definition :
If f : A  B is one to one and onto function and g is a rule under which for every
element Y  B, There exists an unique element X  A then g : B  A is called Inverse
function of f : A  B .
i.e g  f 1

x  g(y)  x  f 1 (y)

So y  f(x) & x  g(y) such that f {g(y)}  y & x  g(f(x)) then f & g are said to be inverse
function of each other.
(i) f 1(x)  [f(x)]1
(ii) arc sin x  sin 1 x
(iii) Arc sin x  2n   arc sin x,n  I
(iv). Only one-one onto function has an inverse function.
Si.No. Function Domain Range
1. sin x x R y  [ 1,1]
2. cos x x R y  [ 1,1]
3. tan x x  R  (2n  1)  2;n  I y R
4. cot x x  R  n;n  I y R
5. cos ec x x  R  n;n  I y  (, 1]  [1, )
6. sec x x  R  (2n  1) /2 ; n  I y  (, 1]  [1, )

Domain, Range and Graph of Inverse Trignometric Functions


1. If sin y  x then y  sin 1 x where 1  x  1&   /2  y   /2 y –1
y = sin x
/2
Note : for the graph of y  sin 1 x take the plane mirror
–1 x
image of the portion of the graph of y  sin x on the line 0 1
y  x from x   /2 to x   /2 graph of y  sin1x –/2
2. If cos y  x then y  cos 1 x where 1  x  1& 0  y   graph of y = cos - 1 x
y
 y =cos–1x

/2

–1 x
0 1

3. If tan y  x then y= tan1 x where x  R &   /2  y   /2 graph of y  tan1 x


y y = tan–1 x
/2

x
0
–/2
Page 58 INVERSE TRIGONOMETRIC FUNCTION
4. If cot y = x then y = cot–1 x where x  R & 0  y   graph of y = cot - 1 x

y y =cot x
–1

/2

x
0

5. If sec y  x then y  sec 1 x where x  1& 0  y  , y   /2 graph of y  sec 1 x


y

/2
 

–1
x
0 1

-1
6. cosec y = x then y = cosec - 1 x where x  1 &   /2  y   /2, y  0 graph of y = cosec x

y
/2

–1 0 1 x

–/2

S.NO. Function Domain Range(Principal Value)


-1
1. y = sin x 1  x  1  /2  y   /2

2. y = cos - 1 x 1  x  1 0y

3. y = tan - 1 x x R  /2  y   /2

4. y = cot - 1 x x R 0y

5. y = cosec- 1 x x 1  /2  y   /2, y  0

6. y = sec - 1 x x 1 0  y  , y   /2
Ex.1 Explain the Following
(i) cos  /4 = 1/ 2 & cos(–  /4) = 1/ 2
But cos 1 (1/ 2) =  / 4 & cos 1(1/ 2)   /4
(ii) sin  /6 =1/2& sin5  /6 = 1/2
But sin11/2 =  /6 & sin 11/2  5  /6
1
Sol. Range of cos x is [0, ]

 1  
 cos 1  
 2 4

Similarly range of sin1 x is (/2, /2)


Page 59 INVERSE TRIGONOMETRIC FUNCTION

1
 sin1    5/6
2
Properties of Inverse Trigonometric Functions
Property I :
(i). sin1 ( x)   sin1 x, x   1,1

(ii). tan1( x)   tan1 x  x  R


(iii). cosec 1( x)   cosec 1 x  x  (, 1]  [1, )

(iv). cos 1 ( x)    cos 1 x  x   1,1

(v). cot1( x)    cot 1 x  x  R


(vi). sec 1( x)    sec 1 x  x  (, 1]  [1, )

 3
Ex.2 Find the principal value of sin 1    .
 2 

1
 3 3    1
 3 
Sol. Let sin   2     sin    2 and  2  sin   2    3 ,sin   2    3
   

 1  1  
Ex.3 Find the value of sin cos     .
  2 

 1  1    1 1 
Sol. sin cos      sin    cos  ( cos 1 ( x)    cos 1 x)
  2   2

   3
 sin      sin  .
 3  3 2
Ex.4 Find the principal value of cot–1(–1).
Sol. Let cot 1 (1)    cot   1 and 0    

3 3
  ,  cot 1(1) 
4 4

  3 
Ex.5 Find the value of sin  tan 1
 3 + 
cos 1
 
 2   .

   

    
Sol. sin tan 1  3  cos 1   3    sin   tan1 3    cos 1 3 
   2  2 
    

 tan1( x)   tan1 x and cos 1( x)    cos 1 x 

   
 sin        sin  1
 3 6 2
Page 60 INVERSE TRIGONOMETRIC FUNCTION

DRILL - I
1. Find the simplified value of each of the following inverse trigonometric terms :

 1  3  2 
(i) sin–1    (ii) cos–1  
 (iii) cosec–1   

 2  2   3

 1
(iv) sec–1 (– 2) (v) cos–1    (vi) cot–1(– 3 )
 2
(vii) cos–1(–1) (viii) tan–1(1) + cos–1(–1/2) + sin–1(–1/2)
1 
(ix) sin–1    + tan–1(1) + cos–1cos   
 2  2

(x) cos–1(–1) – sin–1(1)

 2  1  1  1 
(xi) sin1 
 2 
 cos 1 
 2 
1

  tan  3  cot  

3

 
2. Find the simplified value of the following expressions :

 1  1   1 1 1  1 
(i) sin   sin    (ii) tan cos 2  tan  
 3  2    3 

  1  3    x  1 
(iii) sin–1  cos sin     (iv) sin   sin1    
   2    6  2 

  3    3  
 sin 1
 cos cos 1   
(v) 
sin 2  2  
 (vi)  2  6 
       

  1  2 
(vii) tan  cos   7    /2 
    
3. Draw the graph of the following functions :
(i) y = sin–1 (x + 1) (ii) y = cos–1 (3x) (iii) y = tan–1 (2x – 1)

4. Solve the following inequalities :


(i) sin–1 x > – 1 (ii) cos–1 x < 2 (iii) cot–1 x < – 3

 Property II :
1. y=sin(sin -1 x)= x, x  [- 1 , 1], y  [- 1 , 1], 2. y =cos(cos -1x)= x, x  [- 1 , 1],
y is aperiodic y  [– 1 , 1], y is aperiodic
Page 61 INVERSE TRIGONOMETRIC FUNCTION

3. y =tan (tan–1x)=x, x  R, y  R, 4. y = cot (cot–1x)= x, x  R, y  R,


y is aperiodic y is aperiodic

5. y =cosec(cosec–1x)= x, | x |  1, | y |  1 6. y =sec(sec–1x)= x, | x |  1, | y |  1
y is aperiodic y is aperiodic

 Property III :
   
(i) y  sin1  sin x  , x  R, y   , 
 2 2

  
 2n  x, 2n  2  x  2n  2 ,n  I
1
y = sin (sin x ) = 
(2n  1)  x, (2n  1)    x  (2n  1)   ,n  I
 2 2

Graph of y sin1(sin x)

Period of sin1(sin x) is 2
(ii) y=cos -1(cos x)=x, x  R, y  [0,  ]
 2n  x, 2n  x  (2n  1), n  I
y = cos - 1 (cos x ) = 
2n  x, (2n  1)  x  2n, n  I
Page 62 INVERSE TRIGONOMETRIC FUNCTION

Period of y  cos 1(cos x) is 2

  
(iii). y = tan-1(tan x) = x, x  R- (2n  1) n  I , y    , 
 2   2 2
 
tan1(tan x)  n  x , where n   x  n  n  I
2 2
graph of y  tan 1 (tan x)

1
Period of tan (tan x ) is 
(iv). y = cot -1(cot x) = x, x  R-{n  }, y  (0,  ) graph of y  cot 1 (cot x )

Period of cot1 cot x is 

       
(v) y=sec-1(sec x)=x, y is periodic ; x  R- (2n  1) n  I , y  0,    ,  
 2   2  2 

Period of sec 1(sec x) is 2

    
(vi) y = cosec-1(cosec x), x  R-{n  , n  I}, y   – ,0    0, 
 2   2
graph of y = cosec - 1 (cosec x)

Period of cosec 1(cosec x) is 2


 Property IV :

1
(i). sin1    cosec 1 x;  x  (, 1]  [1,  )
x

1
(ii). cos 1    sec 1 x;  x  (, 1]  [1, )
x
Page 63 INVERSE TRIGONOMETRIC FUNCTION

cot 1 x, x  0
(iii). tan-1(1/x) =  1
   cot x, x  0

1  5π 
Ex.6 Find the principal value of sin sin .
 6 

5  
Sol. sin is to be written as sin  , where     .
6 2 2

 5  1     1   
 sin1  sin  =sin sin       sin  sin  
 6    6   6 6

1  7π 
Ex.7 Find the value of cos  cos .
 6 

Sol. Let cos 1  cos 7     cos   cos 7  cos        cos    3


 6  6  6 6 2

3
cos    and 0    
2
 5
     .
6 6

-1   3  
Ex.8 Find the value of tan  tan   .
  4 

1   3   1     1   1 1 
Sol. tan  tan     tan  tan       tan   tan   tan (1)   tan 1  
  4    4   4 4

tan 1
( x)   tan1 x  .

 -1 3 
Ex.9 Find sin  cos 
 5

3
Sol. Let cos -1 = θ . Then,
5
3 4
cos    sin  
5 5

4
 sin(cos 1 3 /5)  sin  
5

 -1 3 
Ex.10 Find cos  tan 
 4

 1 3 
Sol. Let, cos  tan    . Then,
 4

3 4
tan    cos  
4 5

 cos(tan1 3 /4)  cos   4 /5 tan   3 / 4  cos   4 /5 


Page 64 INVERSE TRIGONOMETRIC FUNCTION

DRILL - II
1. Evaluate the following expressions :

 1 1 
 1 41   1 65 
(i) tan  cos  (ii) cosec  sec 
5  (iii) tan  cos ec 
 3   63 

 1 1   1 4 2 FG 1 IJ
H K
1
(iv) sin   cos  (v) cos  sin  cos 1  (vi) tan cot a
6 4  5 3

12   1  2 
(vii) sin  cos 1  (viii) tan  cos   7    /2 
 13     

  1    
(ix) sec  tan tan   3  
    
2. Evaluate the following inverse trigonometric expressions :
 7   2 
(i) sin–1  sin  (ii) tan–1  tan 
 6   3 

 5   7 
(iii) cos–1  cos  (iv) sec–1  sec 
 4   4 

  17  
(v) cos–1 cos     (vi) sin–1 (sin 10)
  15  

 2   2   5
(vii) cos–1  cos  + sin–1  sin  (viii) tan–1 tan  7 
 3   3   

 4 
(ix) cos 1  cos  (x) cos 1  cos 4 
 3 

 33 
(xi) sin1  cos 
1 1
(xii) cos  cos12  sin  sin12 
 5 

 1  17  
(xiii) sin cot  cot 
  3  
3. Find the value of the following inverse trigonometric expressions :

(i) sin–1 (sin 5) (ii) cos–1 (cos 10)

(iii) tan–1 (tan (– 6)) (iv) cot–1 (cot (– 10))


 1  9 9  
(v) cos–1   cos  sin  (vi) sec2(tan–1 2) + cosec2(cot–1 3)
 2 10 10 

1 1
 5  2 1 1  2 1 1 
(vii) tan  2 cos  3  (viii) sin  cos   cos  sin 
    2  3

(ix) tan2 (sec 1 2)  cot2 (cos ec 1 3)

 Property (V) :

(i). sin1x  cos 1x  , x   1,1
2
Page 65 INVERSE TRIGONOMETRIC FUNCTION

(ii). sec 1 x  cosec 1 x  , x  (, 1]  [1,  )
2
1 1 
(iii). tan x  cot x  , x  R
2
-1 
Ex.11 If sin x = for some x   1,1 , then find the value of cos–1x.
5

1    3
Sol. cos x   sin1 x    .
2 2 5 10
Ex.12 If cos 1x - sin1x = 0 , then find x.

1 1  
Sol. cos x  sin x  0   sin1 x  sin1 x  0   2sin1 x  0
2 2
 1
 sin1 x  x .
4 2

1  3
Ex.13 Find the value of cos 1    + sin1    .
 2 
 2 

2    
Sol.    
3  3 3

DRILL - III
1. Find the value of following expressions :

(i) cot (tan–1 a + cot–1 a) (ii) sin (sin–1x + cos–1x) , | x |  1


 1  3  1  3  1 
(iii) tan  cos    sin    sec 3
 4 4 
2
2. If sin–1 x + sin–1 y =
. Then find the value of cos–1 x + cos–1 y
3
3. Find the minimum and maximum value of
2 2 3 3
(i) sin x   cos x 
1 1
(ii) sin x    cos
1 1
x

Properties VI :

xy
tan1 , x  0, y  0 and xy  1
 1  xy
(a) –1
tan x + tan y = –1
 xy
   tan1 , x  0, y  0 and xy  1
1  xy

xy
(b) x > 0 & y > 0; tan–1x – tan–1y = tan–1 (with no other restriction)
1  xy
Proof :
(a) Let tan–1x = A and tan–1y = B, where A, B  ( /2,  /2)

tan A  tanB xy


Now, tan(A + B) = 1  tanA tanB = 1  xy
Page 66 INVERSE TRIGONOMETRIC FUNCTION

 xy 
 tan1  1
  tan tan(A  B)
 1  xy 

 tan1 tan , where   (, )

 xy 
tan 1  1 
  tan (tan  ) =    ;
 1  xy 

;  /2     / 2
/2    

 tan1 x  tan1 y ;  / 2  tan1 x  tan1 y   / 2


 1 1
tan x  tan y   ;  / 2  tan1 x  tan 1 y  

Case - I :  /2  tan1 x  tan1 y   /2  xy  1 & x, y > 0

 
 A  B  , x.y < 1
2 2

 
tanA . tanB < 1, tanA < cot B, tanA < tan   B 
2 

 
A  B, A  B 
2 2
Case II :  /2  tan1 x  tan1 y    x, y  0 & x.y > 1
Also, tan–1x >  /2 – tan–1y  tan–1x > tan–1(1/y)  x > (1/y)  xy > 1
Remark : If x1, x 2 , x 3 ,...., x n  R , then

 s  s3  s 5 ... 
tan1 x1  tan1 x 2  .....  tan 1 x n  tan1  1 
 1  s2  s 4  s 6 ... 

Where s1  x1  x 2  .....  x n   x1

s 2  x1x 2  x 2 x 3  .....  x n 1 x n   x1x 2

s3  x x 1 2 x 3 ..... and so on.

1 2
Ex.14 Find tan1  tan1 .
4 9

 1 2 
  
Sol. tan1  tan1  tan1  4 9   tan1 1
1 2
4 9 1  1 . 2  2
 4 9
Page 67 INVERSE TRIGONOMETRIC FUNCTION
Ex.15 In a ΔABC,if A = tan1 2and B = tan1 3 then find C.
Sol. Since A, B, C are angles of a triangle, A  B  C    C    (A  B)

1 1 1  2  3  1 1
Now, A  B  tan 2  tan 3    tan      tan (1)    tan 1
 1  2.3 

 1 1 1  x  y  
 tan x  tan y    tan   , x.y  1
  1  xy  

 3 3 
    C    (A  B)    
4 4 4 4

1 1 41
Ex.16 Find cot 9  cosec
4

1 1 41 41  cosec 1 x  cot 1 x 2  1 
Sol. cot 9  cosec  cot 1 9  cot 1 1
4 16  

 1 4 
 95 
1 1 5 1 1 1 4  tan1
 cot 9  cot  tan  tan  
4 9 5 1 1 . 4 
 9 5

 41  
 tan1  1
  tan 1  .
 41  4
Property VII :



sin1 x 1  y 2  y 1  x 2  if x  0; y  0 and x 2  y 2  1

(a) sin–1x + sin–1y = 




  sin1 x 1  y 2  y 1  x 2  if x  0; y  0 and x 2  y 2  1

(b)  2 2

sin–1x – sin–1y = sin–1 x 1  y  y 1  x , x > 0; y > 0

Proof :
(a) Let sin–1x = A and sin–1y = B, where x  0 and y  0
 A, B  [0,  /2]  A  B  [0, ]

Now, sin(A + B) = sinA cosB + sinB cosA = x 1  y 2  y 1  x 2

 
sin1(sin(A  B))  sin1 x 1  y 2  y 1  x 2 
 
sin1 x 1  y 2  y 1  x 2   AB ; 0  A  B  ( / 2)
  (A  B) ; ( / 2)  A  B  

Now, A  B  ( /2)  A  ( /2)  B

 sin A  cosB  x  1  y 2  x 2  y 2  1


and A  B   x2  y 2  1
2
Page 68 INVERSE TRIGONOMETRIC FUNCTION

-1 1 2
Ex.17 If sin + sin-1 = sin-1x , then find x.
3 3
1 2
Sol. We have, sin 1  sin 1  sin 1 x
3 3
1 4 2 1 1 5 2 8 
 sin 1  1  1    sin1 x  sin1  .  . 1
  sin x
3 9 3 9 3 3 3 3 

 5 4 2
 x    .
 9 
 Property VIII :

 
cos–1x + cos–1y = cos–1 xy  1  x 1  y , x > 0, y > 0, x < y
2 2

Ex.18 If cos-1x + cos-1 y + cos-1z =  , 1  x, y, z  1 then find x2 + y2 + z2 + 2xyz.

Sol. Let cos 1 x  ,cos 1 y   and cos 1 z  

 cos   x,cos   y and cos   z . Given       


 cos(  )  cos(   )
 cos  cos   sin  sin    cos 

 xy  1  x 2 . 1  y 2  z

 xy  z  1  x 2 1  y 2

 x 2 y 2  z 2  2xyz  1  x 2  y 2  x 2 y 2 (squaring both sides)

 x 2  y 2  z 2  2xyz  1 .
2 2
1  p  1  q  p q 2
Ex.19 If cos   + cos   =  , then 2 + k cosα + 2 = sin α , where k = ?
a
  b
  a b

  p2   q2  
1 p q pq  p2   q2 
Sol. cos  .   1   1        1    1    cos 
 a b  a2   b2   ab  a2   b2 

2
 pq  p2 q 2 p2 q 2 p2 2pq q2
  cos    1  2  2  2 2  2  cos   2  1  cos 2   sin2 
 ab  a b a b a ab b

2pq
k   .
ab

DRILL - IV
1. Prove that
3  8  77 4 12 33
(i) sin–1   + sin–1   = sin–1 (ii) cos–1 + cos–1 = cos–1
5
   17  85 5 13 65
 1  
(iii) sin–1   + cot–1 3 =
 5 4
Page 69 INVERSE TRIGONOMETRIC FUNCTION

 1  1  1   1
(iv) tan–1  3  + tan–1  5  + tan–1  7  + tan–1  8  =
        4
(v) cot [tan–1(1/7) + tan–1(1/13)] = 9/2
 xy  1  1  yz  1  1  zx  1 
(vi) cot 1    cot    cot  0
 xy   yz   zx 

(vii) cot tan 1 1  tan 1 1  tan 1 1   1


 2 5 8 
 Property IX : (1) 2sin1 x (2) 2cos1 x (3) 2 tan 1 x

 1 1 1 –
2sin x if x n
–1 x 2s
 2 2 2si in –1
x
 1 – –1 11
(1). 
sin1 2x 1  x 2  
1
    2sin x, if
2
 x 1 –2
sin –
x1
 1 1
   2sin x if  1  x 
 2

1
graph of y  sin 2x 1  x
2
 
y
–1
2 –2cos x –1
1
2cos x
2cos x if 0  x  1
(2). cos 1(2x 2  1)   x
1 –1
2  2cos x if  1  x  0 1

graph of y  cos 1(2x 2  1)

–1
–2tanx
2tan1 x if  1  x  1
2x  –1 1
(3). sin1 2
   2tan1 x if x  1
1 x  1
– –2tan– 1x –1
n
x
2 ta
  2tan x if x  1 x

y
–2
tan –1 2tan– 1 x
2
1
2 tan x if x  0 x
1 x
(4). cos 1   1
1  x 2  2tan x if x  0
0 x

1 1  x2
grapf of y  cos
1  x2
Page 70 INVERSE TRIGONOMETRIC FUNCTION
y

2 tan1 x, if  1  x  1 –1
an x
2x   +2t –1
(5) tan 1     2 tan1 x, if x  1 x
2 x
–1
1 x  n 0 –1
1
ta ta n x
   2tan x if x  1 2 – +2

 Property X : (1) 3sin 1 x (2) 3cos 1 x (3) 3 tan 1 x


 1 1 1
3sin x ; 2  x  2

1
(1) sin1  3x  4x 3     3sin 1 x;  x 1
 2
 1 1
  3sin x;  1  x  2

 1 1
2  3cos x ;  1  x  2

(2) cos 1(4x 3  3x)  2  3cos 1x ; 1  x  1
 2 2
 1 1
3cos x ; 2  x  1

graph of y  Cos 1(4x 3  3x)
y

y y=/2
 1 1
  3 tan x, x 
–1 x
x
–1

–1 x n
an 3ta
an

3 3t –+
3t

 y= y=
+

3
1  3x  x 
 3 tan1 x, 1  x  1
y=

tan 
 1 
 ,0 
 1 
 ,0  x
(3) 2   3   3 

 1  3x   3 3
 1 y=–/2
1
  3 tan x, x 
 3

3
1  3x  x 
graph of y  tan  1  3x 2 
 
Page 71 INVERSE TRIGONOMETRIC FUNCTION
 Property XI : Conversion.
(i). for x > 0

 x  1
 1  x2   1  1
sin1 x  cos 1 1  x 2  tan1    cot    sec 1    cosec 1  
2  x  2
 1 x  x
 1 x   
(ii). for x > 0

 1  x2   
x 1  1  1  1 
cos 1 x  sin1 1  x 2  tan1   = cot 1    sec    cosec  
 x  2
x 2
   1 x   1 x 

(iii). for x > 0

 x   1  1  1+ x 2 
tan1 x  sin1 
2
 1 x 
  cos 1 
2
 1 x 
 = cot-1   = sec-1
x
 
1+ x 2 = cosec -1 

 x



Ex.20 Solve 2 tan–1 (cos x) = tan–1 (2 cosec x).
Sol. Given equation is 2 tan–1 (cos x) = tan–1 (2 cosec x)
Let 2tan1  cos x     cos x  tan  /2

2 2 2
Low 2cos ec x   
sin x 2
1  cos x 1  tan2  /2
Given equation becomes

 
   2 
2tan1  tan   tan1  
 2  2  
 1  tan 2 
 

 
 2  2
  tan1    tan  
 2   
 1  tan 2  1  tan2
  2

2tan  /2 2
  tan  /2  1  tan2  /2
 2
1  tan   /2 2
1  tan   /2 

 tan2  /2  1  tan2  /2  2tan2  /2  1

 1   
 tan
2

2  tan 2  0 

1
 cos x   x  /4
2

1 
Note : cos x   cos x  cos
2 4


 x  2n  ,nZ
4
Page 72 INVERSE TRIGONOMETRIC FUNCTION

 5 7
x   , , ,etc.
4 4 4
But for equation (1) to be satisfied cosec x and cos x must have same sign.
 x lies in 1st quadrant

1 1 x 1
Ex.21 Solve tan  tan1 x  x  0 
1 x 2

1 1  1  x 
Sol. Given tan x  2tan  
1  x 

1  x 
2 
 tan1 1  x  2x 
 1 1
2 tan x  tan 1  x 2 
2
1 x 
1  
1  x 

2 1  x 1  x  2 1  x 2 
 tan1 2 2  tan 1
1  x   1  x  4x

1  x2
 tan1 x  tan1
2x

1  x2
 x [ tan–1 x is a one-one function]
2x
 2x2 = 1 – x2  3x2 = 1
1 1
 x  x [ x > 0]
3 3
Ex.22 Write the following in the simplest form

 3a 2 x  x 3  a a
(i) tan1  3 2 , a  0 ;  x
 a  3ax  3 3

 cos x  sin x   3
(ii) tan1  ,   x 
 cos x  sin x  4 4
Sol. (i) Put x = a tan 

a a
  x and a  0
3 3

a a 1 1  
   a tan     tan      .... (1)
3 3 3 3 3 3

x x
 x  a tan   tan      tan1 .... (2)
a a
2 3 2 3 3
1  3a x  x  1  3a a tan   a tan  
Now tan  3 2
 tan  3 2 2 
 a  3ax   a  3a a tan  
Page 73 INVERSE TRIGONOMETRIC FUNCTION

 a 3  3 tan   tan3   
1
 tan    tan1  tan 3 
 a 3
1  3tan 2
  
x
  3   3 tan1 [From (2)]
a

 cos x  sin x  1  1  tan x 


(ii) tan1    tan  
 cos x  sin x   1  tan x 

  
 tan 4  tan x 
1 1  
 tan    tan tan   x     x

 1  tan .tan x  4  4
 4 

1
 1  x 2  1  x2   1 1 2
Ex.23 Prove that tan     cos x
 1  x  1  x  4 2
2 2

 1  x2 
1  
tan1  1  x2 
Sol. L.H.S. =  1  x2  ............. (1)
1  
 1  x2 

1  x2 1  cos 2 2sin2 
Put x 2  cos 2, then 2
   tan   tan 
1 x 1  cos 2 2cos 2 

 1  x2 
 0  2
1  0    
 1 x 4 

1  1  tan  
Now from (1), L.H.S.  tan  
 1  tan  

     1
 tan1  tan           cos 1 x 2
 4  4 4 2

 1 1 2 
 cos 2  x  2  cos x    2 cos x 
2 1 2

DRILL - V
Solving equation involving inverse trigonometric function

 1 1  2x  1 1  1  y 2 
1. Solve tan  2 sin  1 x 2   2 cos  2   , if x > y > 1.
     1  y 
2. Solve sin–1 (cos(sin–1x)) + cos–1 (sin (cos–1x))
1
3. Solve cos (2 sin–1x) = ,
3

4. Solve cot–1 x + tan–1 3 =
2
Page 74 INVERSE TRIGONOMETRIC FUNCTION

 x 1   x 1   2
5. Solve : (i) tan–1   + tan–1   = (ii) sin–1x + sin–12x =
 x  2   x  2  4 3
6. Solve :
1 x  1
(i) tan–1   = tan–1x , (x > 0)
 1  x  2

 1   1  1
(ii) 3tan–1   – tan–1  x  = tan–1  3 
2 3     
7. If 3 cos–1(x2 – 7x + 25/2) =  , then x = ?
–1
8. Solve cos[tan–1{sin (cot x)}]

Summation of series containing inverse trigonometric terms


Ex.24 Find the sum to the n term of the series

cosec–1 10 + cosec–1 50 + cosec–1 170 + . . . + cosec–1 (n2  1)(n2  2n  2)

Sol. Let  = cosec–1 (n 2


 1 )( n 2  2 n  2 )

cosec2  = (n2 + 1) (n2 + 2n + 2)


= {(n2 + 1)2 + 2n (n2 + 1) + n2} + 1 = (n2 + n + 1)2 + 1
cot2  = (n2 + n + 1)2

1 (n  1)  n
tan  = 2 = 1  (n  1)n
n  n 1

 (n  1)  n 
 = tan–1 1  (n  1)n 
 
= tan–1 (n + 1) – tan–1 n
Thus, sum n terms of the given series
= (tan–1 2 – tan–1 1) + (tan–1 3 – tan–1 2) + (tan–1 4 – tan–1 3) + . . . + (tan–1 (n+1) – tan–1 n)

 tan1(n  1)   / 4
Ex.25 Find the sum of the first n terms of the series
cot–1 (3) + cot–1(7) + cot–1(13) + cot–1(21) + . . .
Sol. Let tr denote the rth term of the series 3, 7, 13, 21, . . . and
S = 3 + 7 + 13 + 21 + . . . + tn
Also S = 3 + 7 + 13 + . . . + tn–1 + tn
Subtracting we get
0 = 3 + (4 + 6 + 8 + ..... (n – 1) terms) – tn
n 1
tn = 3 + (2.4 + (n – 2).2)
2
= 3 + (n – 1).(n – 2)
= n2 + n + 1

 1 
Let Tr = cot–1 (r2 + r + 1) = tan–1  2 
 r  r  1

 r 1 r 
= tan–1  1  r(r  1)  = tan–1 (r + 1) – tan–1 r
 
Page 75 INVERSE TRIGONOMETRIC FUNCTION

Thus, the sum of the first n terms of the given series is


(tan–1 2 – tan–1 1) + (tan–1 3 – tan–1 2) + (tan–1 4 – tan–1 3) + . . . + (tan–1 (n+1) – tan–1 n)

 n 1 1   n 
tan–1 (n + 1) – tan–1 (1) = tan–1 1  1(n  1)  = tan–1  
  n 2

DRILL - VI
Solve the following Questions
1. Find the sum of each of the following series :
1 1 1 1
(i) tan-1 x 2  x  1 + tan -1 x 2  3 x  3 + tan -1 x 2  5 x  7 + tan -1 x 2  7 x  13

........ upto n terms.


1 2 2n1
(ii) tan-1 + tan-1 + ..... + tan-1 + ........... 
3 9 1  22n 1

1 2  1 n  n  1
(iii) sin-1 + sin-1 + ..... + sin-1 + ............ 
2 6 n (n  1)
n
1 1 n
1
2. Find the value of  tan 3. Find the value of  tan 1
x 1  x  x  1
2
x 1 2x 2

n
1  2 3
4 Find the value of  cot
x 1
x  
 4

SOLVED EXAMPLES
2
1 2p 1 1  q 2x pq
Ex.1 If sin  cos  tan1 2 then prove that
x
1 p 2
1 q 2
1 x 1  pq .
2
1 2p 1 1  q 2x
Sol. sin 2
 cos 2
 tan1
1 p 1 q 1  x2
 2tan–1 p–2 tan–1 q = 2 tan–1 x
 tan–1 p – tan–1 q = tan–1 x
pq 1
 tan –1 1  pq  tan x

pq
x
 1  pq .
2
Ex.2 If the quardratic equation ; 4 sec 
x 2  2 x  (  2    1 / 2)  0 have real roots, then find all
the possible value of cos   cos 1  .
Sol. Equation have real roots
sec 2
  2 1
 b2  4ac  0  4  4.4       0 ,
 2
2
 1
 4tan 
4((  1/ 2) 2  1/ 4)
Page 76 INVERSE TRIGONOMETRIC FUNCTION
which is possible only when
tan 2
 1
 4 
4((  1/2)2  1/ 4) = 1

1
   0,  &   etc.
2
2
 
Ex.3 Solve the inequality :   sin 1  1  x 2    /3
2 1  x 
2
  1  1  x  
Sol.    sin  2 

3 2 1  x  3

  1  x2  
  cos 1  2 

3 1  x  3
Put x  tan 
   
  2    tan1 x 
3 3 6 6

Ex.4 Solve sin [cot–1 (1 + x)] = cos (tan–1 x).

Sol. sin [cot–1 (1 + x)] = cos (tan–1 x)

 1   1 
 sin sin –1   cos cos 1 
 2  2x  x 2   1  x2 
1 1
 
2
2  2x  x 1  x2
1
 x
2
 n2  10n  26  
Ex.5 If cot 1    ,n  N , then find the minimum value of n .
 2 3  6

n2  10n  26
Sol.  3 minimum value of n  3
2 3

1  x 2  1  x2
Ex.6 If tan –1   , then prove that x 2  sin 2 .
1  x2  1  x 2
Sol. From the given question

1  x2  1  x2 sin 
 tan  
2
1 x  1 x 2 cos 
Apply componendo and dividendo

1  x2 sin   cos 

1 x 2 sin   cos 

1  x 2 1  sin 2
Squaring  , ( cos2   sin 2   1 and 2sin  cos   sin 2 )
1  x 2 1  sin 2
Hence x2 = sin 2 .
Page 77 INVERSE TRIGONOMETRIC FUNCTION

1        sin  cos  
Ex.7 Prove that 2 tan tan tan     = tan 1  
 2  4 2   sin   cos  
1 1 2x
Sol. We know that 2 tan x  tan
1 x2

1 2tan( /2)tan( / 4   /2)
 L.H.S. = tan 1  tan2 ( /2)tan2 ( / 4   /2)

1 2.sin( /2) sin( / 4   /2) cos( /2) cos(  /4   /2)


= tan cos2 ( /2)cos 2 ( / 4   /2)  sin2 ( /2)sin2 ( /4   /2)

1 sin .sin( /2  )
= tan 2[cos( /2   /2   /4)cos( /2   /2   / 4)]

1 sin  cos 
= tan .
cos   sin 

1
Ex.8 Find the value of sin(2 tan–1 ) + cos (tan–1 2 2 ).
3
1 1
Sol. Let tan–1 =  and tan–1 2 2 =  . Then tan  = and tan  = 2 2 , so that
3 3
1
sin (2 tan–1 ) + cos(tan–1 2 2 )
3

2tan  1
= sin 2  + cos  = +
1  tan2  1  tan2 

1
2.
3 1 2 9 1
= 1 + = . +
1 1 8 3 10 3
9

3 1 14
= + =
5 3 15
Ex.9 If x1, x2, x3, x4 are roots of the equation x4 – x3 sin 2  + x2cos 2  – xcos  – sin  = 0,
4
1
then find  tan
i 1
xi .

Sol.  x = sin
1 2 , x x 1 2 = cos 2  ,

x x x =
1 2 3 cos  and x1 x2 x3 x4 = – sin 
 tan–1 x1 + tan–1x2 + tan–1x3 + tan–1x4

 x x x x1 1 2 3

= tan–1  
1   x x  x x x
1 2 1 2 3x 4 

 sin 2  cos    cos (2sin   1) 


= tan–1   = tan–1  sin (2sin   1) 
 1  cos 2  sin    
= tan–1 cot 
Page 78 INVERSE TRIGONOMETRIC FUNCTION

FG  IJ
= tan–1 tan 2   H K

= 
2
Ex.10 Prove that :
 c1x  y   c 2  c1   c3  c2  1 x
tan–1  c y  x  + tan–1  1  c c  + tan–1  1  c c  + .... + tan–1  c  = tan–1 y
 1   2 1  3 2   n
Sol. We have
 c1x  y   c 2  c1   c3  c2  1
tan–1  c y  x  + tan–1  1  c c  + tan–1  1  c c  + .... + tan–1  c 
 1   2 1  3 2   n

 x 1   1 1   1 1 
 y c   c c   c c 
 1
  1 2
  2 3
 1
= tan–1  x 1  + tan–1  1  1  + tan–1  1  1  + ....... + tan–1  c 
1 y c   c1c 2   c 2c 3   n
 1     

x 1 1 1 1 1


= tan–1   –tan–1  c  + tan–1  c  – tan–1   + tan–1   – tan–1  
y  1  1  c2   c2   c3 

F 1I F 1I F 1I FG x IJ
+ tan–1 GH c JK + GH JK
... – tan–1 c + tan–1 c n GH JK = tan–1 y H K
3 n

SINGLE CORRECT TYPE QUESTIONS

Definition, graphs and fundamentals


 3  1 
1. The value of sin–1  2  + sin–1   is equal to
   2
5 3
(A) 75° (B) 105° (C) (D)
12 5
2. Domain of f(x) = cos–1 x + cot–1 x + cosec–1 x is
(A) [– 1, 1] (B) R (C) (–  , - 1]  [1,  ) (D) {– 1, 1}
3. Range of f(x) = sin–1 x + tan–1 x + sec–1 x is
  3    3    3 
(A)  ,  (B)  4 , 4  (C)  ,  (D) none of these
4 4  4 4 
4. cosec–1 (cos x) is real if
(A) x  [– 1, 1] (B) x  R

(C) x is an odd multiple of (D) x is a multiple of 
2
5. If cos [tan  1 { sin (cot  1 3 )} ] = y, then

4 2 2 10
(A) y = (B) y = (C) y = - (D) y2 =
5 5 5 11
6 If x satisfies the equation t2  t  2  0 , then there exists a value for
(A) sin–1x (B)cos–1x (C)sec –1x (D) sin–1 2x
Page 79 INVERSE TRIGONOMETRIC FUNCTION
7 If a  sin1 x  cos 1 x  tan1 x  b , then

 3    3
(A) a   ,b  (B) a  0, b  (C) a  ,b   (D) a  ,b 
4 4 2 2 4 4
1 1
8. The set of values of x satisfying sin x  cos x is

 1   1   1 
(A)  1,  (B)  1,  ,1
 2  2   2 

 1   1 
(C)  1,  (D)  ,
 2  2 
9. The complete solution set [tan 1 x]2  8[tan 1 x]  16  0 of , where [ ] is G.I.F., is
(A) (, tan 4] (B) [tan 4, tan 3] (C) [tan 4, tan 5) (D) 

  2x 2  4  
10. sin1  sin  2      3 , then x satisfies
  1 x 
(A) x  1 (B) x  1 (C) x  1 (D) x  1
2
The solution of the inequality  cot x   5 cot x  6  0 is
1 1
11.

(A)  cot 3, cot 2  (B)  , cot 3   cot 2,  

(C)  cot 2,   (D)  cot1,  


1 1 1 1
12. If sin cos sin tan x   1 , where [.] is G.I.F., then x is given by the interval
(A) [tan sin cos 1, tan sin cos sin 1] (B) (tan sin cos 1, tan sin cos sin 1)
(C) [-1, 1] (D) [sin cos tan 1, sin cos sin tan 1]
n 
13. If cot 1      ,n  N , then the maximum value of ‘n’ is
   6
(A) 7 (B) 5 (C) 9 (D) 6
14. If   x  2  , then cos–1 (cosx) is equal to
(A) x (B)  – x (C) 2  + x (D) 2  – x
2
15. If sin–1 x + sin–1 y = , then cos–1x + cos–1y is equal to
3
2  
(A) (B) (C) (D) 
3 3 6
-1 -1 -1
16. If x  0 and  = sin x + cos x - tan x, then
 3    
(A)    (B) 0    (C) 0   < (D)   
2 4 4 2 4 2
1
17. If x < 0 then value of tan–1(x) + tan–1   is equal to
x
 
(A) (B) – (C) 0 (D) none of these
2 2
Page 80 INVERSE TRIGONOMETRIC FUNCTION

 1  3  1  2  
18. The value of tan sin    tan    is
 5  3 
6 7 5 17
(A) (B) (C) (D)
17 16 7 6
–1 –1
19. tan a + tan b, where a > 0, b > 0, ab > 1, is equal to
 ab   ab 
(A) tan–1   (B) tan–1   – 
 1  ab   1  ab 

 ab   ab 
(C)  + tan–1   (D)  – tan–1  
 1  ab   1  ab 

1 1
20. tan–1   + tan–1   is equal to
2 3
  
(A) (B) (C) (D) none of these
4 2 3
3  5 
21. cos–1   + cos–1   is equal to
5  13 

 33   33   64 
(A) cos–1   (B) cos–1    (C) cos–1   (D) none of these
 65   65   65 

 3
22. The equation sin–1 x – cos–1x = cos–1  2  has :
 
(A) no solution (B) unique solution
(C) infinite number of solutions (D) none of these
1 
23. If sin–1x + cot–1   = , then x is equal to
2 2
1 2 3
(A) 0 (B) (C) (D)
5 5 2

   3 
24. The solution of the equation sin-1  tan  – sin-1  x  – = 0 is
 4   6
(A) x = 2 (B) x = - 4 (C) x = 4 (D) none of these
n n
1
25. If  cos
i 1
 i = 0, then 
i 1
i is equal to

(A) n (B) – n (C) 0 (D) none of these


26. The set of values of ‘x’ for which the formula 2 sin –1x = sin–1 (2x 1 x 2 ) is true, is

 3 3  1 1 
(A) (– 1, 0) (B) [0, 1] (C)   2 , 2  (D)   , 
   2 2

1  yz  1  zx  1  xy 
27. If x2 + y2 + z2 = r2, then tan    tan    tan  
 xr   yr   zr 
(A)  (B)  /2 (C) 0 (D) None of these
Page 81 INVERSE TRIGONOMETRIC FUNCTION

28. If cot –1 7 + cot–1 8 + cot–1 18 =  , then cot  is equal to


(A) 1/3 (B) 3 (C) 1/2 (D) 2

 1 1   1 1 
29. tan   cos x  + tan   cos x  , x  0 is equal to
4 2  4 2 
2 x
(A) x (B) 2x (C) (D)
x 2
 
30. The value of sin–1 [cos{cos–1 (cosx) + sin–1 (sin x)}], where x   ,   is
2 
   
(A) (B) (C) – (D) –
2 4 4 2

1  x2  1
-1
31. If tan = 4°, then:
x
(A) x = tan 2° (B) x = tan 4° (C) x = tan (1/4)° (D) x = tan 8°
 1  sin x  1  sin x  
32. The value of cot-1   , where < x <  , is:
 1  sin x  1  sin x  2

x  x x x
(A)  - (B) + (C) (D) 2  -
2 2 2 2 2
33. The number of solution(s) of the equation, sin -1x + cos-1 (1 - x) = sin-1 (- x), is/are
(A) 0 (B) 1 (C) 2 (D) more than 2
 1 x 
34. The smallest and the largest values of tan –1  1  x  , 0  x  1 are
 
    
(A) 0,  (B) 0, (C) – , (D) ,
4 4 4 4 2
35. The complete solution set of the inequality [cot –1x]2 – 6 [cot–1 x] + 9  0, where [.]
denotes G.I.F., is
(A) (–  , cot 3] (B) [cot 3, cot 2] (C) [cot 3,  ) (D) none of these
1  3sin2  
36. If sin-1   = , then tan  is equal to
2  5  4cos 2  4
(A) 1/3 (B) 3 (C) 1 (D) - 1

MULTIPLE CORRECT TYPE QUESTIONS

1 1   14 
1. The value of cos  2 cos cos   5  is:
   

 7      2  3
(A) cos    (B) sin   (C) cos   (D) - cos  
 5   10   5   5 
2. sin–1 x > cos–1 x holds for
 1   1 
(A) all values of x (B) x   0 ,  (C) x   , 1 (D) x = 0.75
 2  2 
Page 82 INVERSE TRIGONOMETRIC FUNCTION

1  x2
3. If 0 < x < 1, then tan–1 is equal to:
1 x

1 1 x 1 x 1 1 x
(A) cos-1 x (B) cos-1 (C) sin-1 (D) tan-1
1 x
2 2 2 2
4. If cos-1x = tan-1x, then
 5 1  5  1
(A) x2 =  2  (B) x2 =  2 
   

 5 1  5 1
(C) sin (cos-1x) =  2  (D) tan (cos-1x) =  2 
   

 4n
5.  tan -1 n4  2n2  2 is equal to:
n 1

(A) tan -1 2 + tan -1 3 (B) 4 tan -1 1

(C)  /2 (D) sec -1


 2 
 2 17 
6. If 6 sin–1  x –6x   =  , then
 2 
(A) x = 1 (B) x = 2 (C) x = 3 (D) x = 4
3
7. If sin–1x + sin–1y + sin–1z = , then
2
9
(A) x100 + y100 + z100 – x101  y101  z101 = 0

(B) x22 + y42 + z62 – x220 –y420 – z620 = 0


(C) x50 + y25 + z5 = 0
x 2008  y 2008  z 2008
(D) 0
(xyz)2009
8. If  satisfies the inequation x 2 – x – 2 > 0, then a value exists for
(A) sin–1  (B) cos–1  (C) sec–1  (D) cosec–1 

COMPREHENSION TYPE QUESTIONS

Passage - I
Let the domain and range of inverse circular functions are defined as follows
Domain Range

sin–1x [–1, 1]   3 
2 , 2 
 
cos–1x [–1, 1] [0,  ]
  3 
tan –1x R  , 
2 2 
cot–1x R (0,  )
Page 83 INVERSE TRIGONOMETRIC FUNCTION

  3 
cosec–1x (–  , –1]  [1,  ) 2 , 2  – {  }
 


sec –1 x (–  , –1]  [1,  ) [0,  ] –  2 
 
3
1. sin–1x < then solution set of x is
4

 1   1   1 1 
(A)  , 1 (B)   , 1 (C)  ,  (D) none of these
 2   2   2 2
2. sin x + cosec–1x at
–1
x = –1 is
(A)  (B) 2  (C) 3 (D) – 
3. If x  [–1, 1], then range of tan–1(–x) is
 3 7    3 5    
(A)  , (B)  , (C) [–  , 0] (D)   , 
4 4  4 4   4 4
4. For x  0,1 ,sin 1 x is equal to


(A) cos 1 1 x 2  1 2
(B) cos 1  x 
2
(C)  cos 1 1  x 2 (D) sin 1 1  x 2
1 1
5. Number of solutions of tan x  cos x  0 is/are
(A)2 (B) 1 (C) 0 (D) 3
6. The root of the equation x  6 x  1  0 is
4 2

   
(A) tan (B) tan (C)tan (D) tan
12 4 8 16

Passage - II

 3   3 
  ,      ,  2     2
2 2 

  , sin–1 (sin  ) =    
tan–1 (tan  ) =   ,    ,   ,
2 2  2 2

  3   3
   ,    , 2

2
 2 2 

  ,     0

 , 0
cos–1 (cos  ) = 
2   ,     2

Based on the above results, answer each of the following :

7. cos–1 x is equal to
(A) sin–1 1  x2 if –1 < x < 1 (B) –sin–1 1  x2 if –1 < x < 0
(C) sin–1 1  x 2 if –1 < x < 0 (D) sin–1 1  x2 if 0 < x < 1
8. sin–1 x is equal to
(A) cos–1 1  x2 if –1 < x < 0 (B) cos–1 1  x2 if –1 < x < 1
(C) cos–1 1  x2 if 0 < x < 1 (D) –cos–1 1  x2 if 0 < x < 1
Page 84 INVERSE TRIGONOMETRIC FUNCTION
9. cos–1 x is equal to

1  x2 1  x2
(A) –tan–1 if –1 < x < 0 (B) tan–1 if –1 < x < 0
x x

1  x2 1  x2
(C) –tan–1 if 0 < x < 1 (D)  + tan–1 if –1 < x < 0
x x
Passage - III
We know that corresponding to every bijection (one-one function) f : A  B , there
exists a bijection g : B  A defined by g ( y )  x if f ( x )  y and only if . The function
g : B  A is called the inverse of function f : A  B and is denoted by f 1 . Thus, we
have f ( x )  y  f 1 ( y )  x . We have also learnt that for all x A and

( fof 1 )  f ( f 1 ( y ))  f ( x )  y , for all y  B . We know that trigonometric functions are


periodic functions and hence, in general all trigonometric functions are not bijectives.
consequently, their inverse do not exists. However, if we restrict their domains and
codomains, they can be mod bijections and we can obtain their inverse.
10. cos 1  cos     , for all belonging to

      
(A) 0,  (B)   ,  (C)   ,   0 (D) 0,     
 2 2  2 2 2 
11. sec 1  sec     , for all  belonging to
(A) 0,    /2 (B)  0,     x /2

 
(C)  0,   (D)  0, 
 2
12. tan1  tan      2 , for all belonging to

 3 5   
(A) R (B)  (C)  ,  (D)  , 
 2 2   2 2
13. The value of sin 1 (sin12)  cos 1 (cos12) , is
  
(A) 0 (B) (C) – (D)
2 2 3
 
The sin cos cos  cos x   sin  sin x  , where x   ,   , is
1 1 1
14.
2 
 
(A) (B)  (C)  (D) 
2 2

MATRIX-MATCH QUESTIONS
1. Column - I Column - II
 1 4 2 17
(A) The value of tan  cos  tan1  is (p)
 5 3 6

 1 1  7
(B) The numerical value of tan  2tan   is (q)
 5 4 17
Page 85 INVERSE TRIGONOMETRIC FUNCTION

1 1 1  3
(C) The value of cos  cos  is (r)
2 8 4

1 5
(D) The value of cos tan– 1 sin cot– 1   is (s)
2 3
2. Column - I Column - II

(A) sin– 1 x – cos– 1x is maximum at (p) x = –1


1
(B) (sin– 1 x)2 + (cos– 1 x)2 is minimum at (q) x = –
2
– 1 2 – 1 2
(C) (tan x) + (cot x) is minimum at (r) x=0
1
(D) (sin– 1 x)3 + (cos– 1 x)3 is maximum at (s) x=
2
(t) x=1
1 1 1 1
sin x cos x sin x cos x
3. Let t1  sin1 x  ,t2  sin 1 x  , t 3   cos 1 x  ,t 4   cos 1 x 
Column - I Column - II
(A) x   0, cos1 (p) t1 > t2 > t4 > t3

 1 
(B) x   cos1,  (q) t4 > t3 > t1 > t2
 2

 1 
(C) x  ,sin1 (r) t2 > t1 > t4 > t3
 2 
(D) x   sin 1,1 (s) t3 > t4 > t1 > t2

4. Column- I Column-II
 1  1  3 5
(A) sin   sin     (p)
3  2  2

 1  3   24
(B) cos  cos  2   6 
  (q)
7
   

1 1 5
(C) tan  2 cos 
3  (r) 1

 1 4
(D) tan  2sin (s) –1
 5 

INTEGER TYPE QUESTIONS

1. Let a, b, c be three positive real numbers


a (a  b  c) b (a  b  c) c (a  b  c) 
 = tan– 1 + tan– 1 + tan– 1 then find .
bc ca ab 
Page 86 INVERSE TRIGONOMETRIC FUNCTION

2. Find the number of real solutions of equation 1  cos 2x = 2 sin–1 (sin x), -   x   .
k
3. If the sum of the series cot–1 2 + cot–1 8 + cot–1 18 + cot–1 32 + ... upto  is then find K.
4

–1  2m  k
4. If  tan
m1
 4 2 
 m  m  2 4
. Find K

–1 xr yr zr
5. If r = x + y + z, & tan  tan–1  tan –1  k then find K
yz zx xy
6. The no. of positive integral solutions (x, y) of the equation tan–1 x + cot–1 y = tan–1 3 are
5 2
7. If (tan–1 x)2 + (cot–1 x)2 = , find |x|
8
x 4  y 4  z 4  4x 2 y 2 z 2
–1 –1 –1
8. If sin x + sin y + sin z =  then find
x 2y 2  y2z2  z 2x2

3 (x101  y101 )(x 202  y 202 )


9. If sin –1
x + sin –1
y + sin –1
z= . find 
2 (x 303  y 303 )(x 404  y 404 )

PREVIOUS YEAR QUESTIONS (AIEEE / JEE MAINS)

1 2
1. tan–1   + tan–1   is equal to - [AIEEE-2002]
 4 9
1 3 1 3 1 3 1
(A) cos 1   (B) sin1   (C) tan1   (D) tan–1
2 5 2 5 2 5 2

2. cot–1  cos   – tan–1  cos   = x. then sin x is equal to - [AIEEE-2002]

  


(A) tan2   (B) cot2   (C) tan  (D) cot  
 2  2  2
–1 –1
3. The trigonometric equation sin x = 2 sin  , has a solution for [AIEEE-2003]
1 1
(A) < | | < (B) all real values of 
2 2
1 1
(C) |  |  (D) |  | 
2 2
y
4. If cos–1x – cos–1 =  , then 4x2 – 4xy cos  + y2 is equal to- [AIEEE-2005]
2
(A) 2 sin 2  (B) 4 (C) 4 sin2  (D) – 4 sin2 
x 5 
5. If sin–1   + cosec–1   = then a value of x is- [AIEEE-2007]
5
  4
  2
(A) 1 (B) 3 (C) 4 (D) 5
 –1 5 2
6. The value of cot  cos ec  tan –1  is [AIEEE-2008]
 3 3
3 2 5 6
(A) (B) (C) (D)
17 17 17 17
Page 87 INVERSE TRIGONOMETRIC FUNCTION

7. If x, y, z are in A.P. and tan–1 x, tan–1 y and tan–1 z are also in A.P., then:
[JEE MAINS-2013]
(A) 6x = 3y = 2z (B) 6x = 4y = 3z (C) x = y = z (D) 2x = 3y = 6z
 2x  1
8. Let tan1 y  tan1 x  tan1  2 
, where | x| . Then a value of y is :
 1  x  3
[JEE MAINS-2015]
3x  x 3 3x  x 3 3x  x 3 3x  x 3
(a) (b) (c) (d) .
1  3x 2 1  3x 2 1  3x 2 1  3x 2

PREVIOUS YEAR QUESTION (IIT-JEE / JEE ADVANCED)


1. The number of real solutions of tan-1 x (x  1) + sin-1 x 2  x  1 = is:
2
[IIT-JEE 1999]

(A) zero (B) one (C) two (D) infinite

 x2 x3   2 x4 x6 
2. If sin 
-1x    .......  + cos  x 
-1   ....... =  for 0 < | x | < 2 , then x
 2 4   2 4  2
equals [IITJEE-2001]

(A) 1/2 (B) 1 (C) – 1/2 (D) –1

x2  1
3. Prove that, cos tan -1 sin cot -1 x = . [IIT-JEE-2002]
x2  2
4. The value of x for which sin ( cot–1 (1 + x)) = cos (tan–1 x) is [IIT-JEE-2005]
(A) 1/2 (B) 1 (C) 0 (D) – 1/2
5. Match the column [IIT-JEE-2007]

Let (x, y) be such that sin–1 (ax) + cos–1 (y) + cos–1 (b xy) =
2
Column – I Column – II
(A) If a = 1 and b = 0, then (x, y) (p) lies on the circle x2 + y2 = 1
(B) If a = 1 and b = 1, then (x, y) (q) lies on (x2 – 1) (y2 – 1) = 0
(C) If a = 1 and b = 2, then (x, y) (r) lies on y = x
(D) If a = 2 and b = 2, then (x, y) (s) lies on (4x2 – 1) (y2 – 1) = 0
–1 –1 2 1/2
6. If 0 < x < 1, then 1  x 2 [{x cos (cot x) + sin (cot x)} – 1] = [IIT-JEE 2008]

x
(A) (B) x (C) x 1  x 2 (D) 1  x2
1  x2

 1  sin      d
7. Let f()  sin  tan    , where     . Then the value of
4 4
(f()) is
  cos 2   d(tan )
[JEE ADV. 2011]

8.
 23
 n 1
1

k
n

1

 
The value of cot   cot 1   2k  is  [JEE ADV. 2013]

23 25 23 24
(A) (B) (C) (D)
25 23 24 23
Page 88 INVERSE TRIGONOMETRIC FUNCTION

9. Match list I with list II and select the correct answer using the code given below the
lists :
Column - I Column-II

2 1/2
 1 1 
 1  cos(tan y)  y sin(tan y)   y 4  1 5
P.  2  1 1  takes value 1.
 y  cot(sin y)  tan(sin y)   2 3
 

Q. If cos x + cos y + cos z = 0 = sin x + sin y + sin z then 2. 2

xy
possible value of cos is
2

  1
R. If cos   x  cos 2x  sin x sin2x sec x  cos x sin2x sec x 3.
4  2

 
 cos   x  cos 2x then possible value of sec x is
4 


S. If cot  sin

1 
 
1  x 2   sin tan1 x 6 , x  0 , then

 4. 1

possible value of x is [JEE ADV. 2013]


(A) P-4, Q-3, R-1, S-2 (B) P-4, Q-3, R-2, S-1
(C) P-3, Q-4, R-2, S-1 (D) P-3, Q-4, R-1, S-2
10. Let f : [0,4]  [0, ] be defined by f(x) = cos–1 (cos x). The number of points x  [0, 4]
10  x
satisfying the equation f (x )  is .......... [JEE ADV. 2014]
10

1  6  1  4 
11. If   3sin   and   3cos   , where the inverse trigonometric functions take
 11  9
only the principal values, then the correct option(s) is(are) [JEE ADV. 2015]
(A) cos  > 0 (B) sin  < 0 (C) cos(  +  ) > 0 (D) cos  < 0
Page 89 INVERSE TRIGONOMETRIC FUNCTION

ANSWER KEY
DRILL-I
   3 2
1. (i) – (ii) (iiii) – (iv) (v) (vi) 5/6
6 6 3 4 3
(vii)  (viii) 3/4 (ix) 7/12 (x) /2 (xi) 17/12
1 
2. (i) 1 (ii) (iii) (iv) 0 (v) 1/2 (vi) –1
3 6
2
(vii)
45

3. (i) (ii)

(iii)

4. (i) – sin 1 < x  1 (ii) cos2 < x  1 (iii) no solution

DRILL - II
41 63 1 3 5
1. (i) 2 2 (ii) (iii) (iv)
4 16 8

64 5 2
(v) (vi) a (vii) 5/13 (viii) (ix) 2
15 3 5
  3 
2. (i) – (ii) – (iii) (iv) (v) 13/15 (vi) 3-10
6 3 4 4
(vii)  (viii) -2/7 (ix) 2/3 (x) 2  – 4 (xi) -/ 10 (xii) 8  24

3
(xiii)
2
17
3. (i) 5 – 2 (ii) 4 – 10 (iii) 2 – 6 (iv) 4 – 10 (v) (vi) 15
20

3 5
(vii) (viii) 59/36 (ix) 11
2

DRILL - III
1   2 5 2 3 73
1. (i) 0 (ii) 1 (iii) 2. 3. (i) , , (ii) ,
2 2 3 8 4 32 8
Page 90 INVERSE TRIGONOMETRIC FUNCTION

DRILL - IV

All Questions are Proved that

DRILL - V
1  xy  1 1 1
1. xy
2. 3. ± 4. x = 3. 5. (i) ± , (ii) x =
2 3 2 2

1 x2 1
6. (i) x = (ii) x = 2 7. 3 or 4 8.
3 x2  2

DRILL - VI
 
1. (i) tan1(x  n)  tan1 x , (ii) , (iii)
4 2

 n   n   4n 
2. tan1   3. tan 1   4. tan1  
 n  2  n 1  2n  5 

SINGLE CORRECT TYPE QUESTIONS


1. B 2. D 3. C 4. D 5. B 6. C 7. D
8. A 9. D 10. B 11. B 12. A 13. B 14. D
15. B 16. D 17. B 18. D 19. C 20. A 21. B
22. B 23. B 24. C 25. A 26. D 27. B 28. B
29. C 30. D 31. D 32. B 33. B 34. B 35. A
36. B

MULTIPLE CORRECT TYPE QUESTIONS

1. BCD 2. C, D 3. ABC 4. AC 5. AD
6. BD 7. AB 8. CD

COMPREHENSION TYPE QUESTIONS


1. A 2. C 3. B 4. A 5. C 6. C 7. D
8. C 9. D 10. A 11. A 12. C 13. A 14. D

MATRIX MATCH QUESTIONS


1. (A)  (p), (B)  (q),(C)  (r),(D) (s) 2. (A)  (t),(B)  (s),(C) (t),(D)  (p)
3. (A)  (q),(B)  (s),(C) (r),(D)  (p) 4. (A)  (r),(B)  (s),(C) (p),(D) (q)

INTEGER TYPE QUESTIONS


1. (1) 2. (2) 3. (1) 4. (1) 5. (1) 6. (2) 7. (1)
8. (2) 9. (3)
PREVIOUS YEAR QUESTIONS (AIEEE)
1. D 2. A 3. C 4. C 5. B 6. D 7. C
8. A
PREVIOUS YEAR QUESTIONS (IIT JEE)
1. C 2. B 4. D 5. (A) (p),(B) (q),(C)  (p),(D)  (s)
6. C. 7. (1) 8. B 9. B 10. 3 11. BCD

You might also like